You are on page 1of 141

Sample Calculus Problems

Part 1: Single Variable Functions

Part 2: Multi-Variable Functions

66

Part 3: Sequences and Series

111

Part 4: Vector Analysis

133

Last revision: September 3, 2015

[This

page is intentionally left blank.

Part 1: Single Variable Functions

1.

Evaluate the limit

Solution:

Remark:

4 3x + 1
lim 2
x5 x 7x + 10

. (Do not use L'Hpital's Rule.)

4 3x + 1
(4 3x + 1)(4 + 3x + 1)

lim 2
= lim
x5 x 7x + 10
x5 (x2 7x + 10)(4 +
3x + 1)
16 (3x + 1)

= lim
x5 (x2 7x + 10)(4 +
3x + 1)
15 3x

= lim
x5 (x 5)(x 2)(4 +
3x + 1)
3(5 x)

= lim
x5 (x 5)(x 2)(4 +
3x + 1)
3

= lim
x5 (x 2)(4 +
3x + 1)
3

=
(5 2)(4 + 3 5 + 1)
3
=
38
1
=
8

 = is the most frequently used verb in mathematics. It was introduced in 1557 by

Robert Recorde to avoid the tedious repetition of the words `is equal to'. It is important to
use the equal sign correctly.

To introduce  , the phantom equal sign, to avoid the tedious repetition of the symbol
 = is not a good idea. The solution above should not go like:

4 3x + 1
lim 2
x5 x 7x + 10

(4 3x + 1)(4 + 3x + 1)

lim
x5 (x2 7x + 10)(4 +
3x + 1)
15 3x

lim
x5 (x 5)(x 2)(4 +
3x + 1)
3(5 x)

lim
x5 (x 5)(x 2)(4 +
3x + 1)

One must also not use other symbols, which have completely dierent meanings, in place
of  =. The solution above should not go like:

4 3x + 1
(4 3x + 1)(4 + 3x + 1)

lim 2
lim
x5 x 7x + 10
x5 (x2 7x + 10)(4 +
3x + 1)
15 3x

lim
x5 (x 5)(x 2)(4 +
3x + 1)
3(5 x)

 lim
x5 (x 5)(x 2)(4 +
3x + 1)

The equal sign always stands between two things, although sometimes one of these things
are at the end of the previous line or at the beginning of the next line. The solution above
should not start like:

This begs the question: What is

(4 3x + 1)(4 + 3x + 1)

= lim
x5 (x2 7x + 10)(4 +
3x + 1)

(4 3x + 1)(4 + 3x + 1)

equal to lim
?
x5 (x2 7x + 10)(4 +
3x + 1)

The equal sign can be used between two functions when we deal with identities , like

x2 1
=x+1
x1

for all

x =/ 1

or when we deal with equations , like

Find all

such that

x2 = 4.

Therefore we can not just drop some of the limit signs in the solution above to make it
look like:

4 3x + 1 (4 3x + 1)(4 + 3x + 1)

lim 2
=
x5 x 7x + 10
(x2 7x + 10)(4 + 3x + 1)

=
(x 2)(4 + 3x + 1)
3

=
(5 2)(4 + 3 5 + 1)
3
=
38
1
=
8

The equalities on the lines marked with


not equal to

1
8

are not correct.

because, for instance, if we let

3
1
1

= =/ .
8
(1 2)(4 + 3 1 + 1) 2
2

x = 1

then

is
(x 2)(4 + 3x + 1)
3

=
(x 2)(4 + 3x + 1)

2.

Let

Express

be the slope of the tangent line to the graph of


as a limit. (Do not compute

Solution:

(x0 , f (x0 ))

The slope

x2
x+2

y =

m.)

at the point

y = f (x)

of the tangent line to the graph of

(3, 9).

at the point

is given by the limit

f (x) f (x0 )
xx0
x x0

m = lim
or equivalently by the limit

f (x0 + h) f (x0 )
.
h0
h

m = lim
Therefore two possible answers are

x2
(3 + h)2
(9)
(9)
x
+
2
3
+
h
+
2
m = lim
= lim
.
x3
h0
x (3)
h

3.

Suppose that

Solution:

lim f (x) =/ 0

and

lim g(x) = 0.

Assume that

lim

f (x)
g(x)

xc

xc

xc

Show that

exists, and let

f (x)
xc g(x)

lim

L = lim
xc

f (x)
.
g(x)

does not exist.

Then by the product

rule for limits we obtain

lim f (x) = lim (


xc

xc

f (x)
f (x)
g(x)) = lim
lim g(x) = L 0 = 0 .
xc
g(x)
g(x) xc

This contradicts the fact that


true:

4.

lim
xc

f (x)
g(x)

lim f (x) =/ 0.
xc

Therefore our assumption cannot be

does not exist.

lim f (x) = 0 and there exists a constant K such that g(x) K


xc
interval containing c. Show that lim (f (x)g(x)) = 0.

Suppose that

some open

Solution:

for all

x =/ c

xc

We have

f (x)g(x) = f (x) g(x) f (x) K

in some open interval

around c. Therefore

Kf (x) f (x)g(x) Kf (x) .


Now applying the Sandwich Theorem and using the fact that
the result.

lim f (x) = 0 we obtain


xc

in

5.

Determine the following limits if

a.

x0

d.

x0

lim f (x) = A

x0+

lim f (x2 x)

b.

x0

lim (f (x3 ) f (x))

e.

x1

and

lim f (x) = B .

x0

c.

lim (f (x2 ) f (x))

lim f (x3 x)

x0+

lim f (x2 x)

Solution: a.

If x < 0, then x2 > 0 and x > 0. Therefore x2 x > 0 for x < 0, and
2
x x approaches 0 from the right as x approaches 0 from the left. lim f (x2 x) = A.
x0

b.

Since

x2 > 0

the left. Hence

c.

For

as

d.

Since

left.

e.
x

6.

Let

(x 1)2

x < 0, x2 approaches 0 from the right as x approaches


lim (f (x2 ) f (x)) = lim f (x2 ) lim f (x) = A B .
for

x0

0 < x < 1,

x0

we have

x3 < x

x0

x3 x < 0. So x3 x approaches
3
Therefore lim f (x x) = B .
+

and

approaches 0 from the right.

0 from the left

x0

x3 < 0 for x < 0, x3 approaches 0 from the left as x approaches


3
3
Hence lim (f (x ) f (x)) = lim f (x ) lim f (x) = B B = 0.

For

x0

0<x<1

x0

we have

x2 < x

approaches 1 from the left.

Q be the point
+ y 2 = 1 and the

0 from

0 from the

x0

x2 x < 0. x2 x approaches
2
Hence lim f (x x) = B .

and

0 from the left as

x1

of intersection in the rst quadrant of the circle C1 with equation


circle C2 with equation x2 + y 2 = r 2 . Let R be the point where the line

passing through the points


as r 0+ .

P (0, r)

and

intersects the

x-axis.

Determine what happens to

Solution:

x2 + y 2 = r2 from (x 1)2 + y 2= 1 we obtain x = r2 /2,


in x2 + y 2 = r 2 gives us Q(r 2 /2,
r2 r4 /4).

Subtracting

substituting this back

and

R(a, 0) be the coordinates of R and let S be the foot of the perpendicular from
Q to the x-axis. Since the triangles RSQ and ROP are similar we have
Let

a r2 /2
a

=
r2 r4 /4 r
and hence

a=

r3 /2

.
r r2 r4 /4

Then

lim+ a = lim+

r0

r0

r3 /2

r r2 r4 /4

r3 /2

(r
+
r2 r4 /4))
r0
r2 (r2 r4 /4)

= 2 lim+ (1 + 1 r2 /4)
r0

= 2 (1 + 1 02 /4) = 4 .
= lim+ (

Therefore

7.*

approaches the point

(4, 0)

as

r 0+ .

Use the formal denition of the limit to show that

Solution:

Given

>0

we want to nd

>0

lim

x1/2

1
= 2.
x

such that

1
1
0 < x < 2 < .
2
x

()

We will do this in two dierent ways.

Solve the Inequality Method : First we solve

1
2 <
x

for

x.

1
1
2 < 2 < < 2 +
x
x
The next step depends on whether
If

2 > 0,

that is if

< 2,

is positive, zero or negative.

then

2<

1
1
1
< 2 +
>x>
.
x
2
2+

*Examples marked red are not part of the Fall 2015 syllabus.

If

2 = 0,

that is if

= 2,

then

2<

If

2 < 0,

that is if

> 2,

2<

1
1
< 2 + x >
.
x
2+

then

1
1
< 2 + x >
x
2+

or

1
>x.
2

Next we choose

in such a way that every

lies in the solution set of

1
2 < ,
x

and therefore the implication

all three cases choosing a delta such that

1
0 < x <
2
in () holds. In

satisfying the condition

0 < 1/2 1/(2 + ) = /(4 + 2)

achieves

this.

The Estimation Method : Suppose

0 < x 1/2 <

for some

> 0.

Then

1
2x 1/2 2
2 =
<
.
x
x
x
to satisfy 1/4. (Why 1/4?) Then
0 < x 1/2 < 1/2 < x < 1/2 + 1/4 < x < 3/4 4 > 1/x > 4/3
1/x < 4 and therefore
1
2
2 <
< 8 .
x
x

At this point let us also decide to choose

> 0 if we choose to satisfy /8 (as well as 1/4) then


1

we will have 2 < 8 8


= and () will hold. In conclusion, any choice of
x
8
satisfying 0 < min{/8, 1/4} works.

Hence for a given

8.* Show that x3


lim (x4 + 7x 17) = 43 using the formal denition of the limit.
Solution:

For any given

>0

we have to nd a

>0

so that for all

0 < x (3) < x4 + 7x 17 43 < .


*Examples marked red are not part of the Fall 2015 syllabus.
7

we have

(x4 + 7x 17) 43 = x4 + 7x 60 = (x + 3)(x3 3x2 + 9x 20). Suppose


that 0 < x (3) < and 1. Then 4 3 < x < 3 + 2. In particular,
x < 4. Therefore, using the Triangle Inequality, we obtain x3 3x2 + 9x 20
x3 + 3x2 + 9x + 20 < 43 + 3 42 + 9 4 + 20 = 168. Now if we choose to satisfy
0 < min{/168, 1}, then we have

x4 + 7x 17 43 = x4 + 7x 60 = x + 3 x3 3x2 + 9x 20 < 168


168 =
168
whenever 0 < x (3) < . We are done.
We have

9.* Suppose that for all 0 < < 1,


x 1 <

2
f (x) 3 <
4

x 1 <

g(x) 4 < .
35

and

Find a real number

>0

If we take

1
10

in

1
10

1
1
f (x) 3 <
.
400
10

() we get

in

x 1 <
Therefore if

1
.
5

() we get

x 1 <
If we take

()

such that

x 1 < f (x) + g(x) 7 <

Solution:

()

1
,
400

x 1 <

1
1
g(x) 4 <
.
350
10

then we have

f (x) + g(x) 7 = f (x) 3 + g(x) 4 f (x) 3 + g(x) 4 <


by the Triangle Inequality. Hence we can take

Remark:

10.*

Let

In fact, any

f (x) =

a.

Show that if

b.

Show that

94 2

1225

if

x=

1
n

works.

where

is a positive integer,

otherwise.

c =/ 0

then

lim f (x)
x0

1
.
400

lim f (x) = 0.
xc

does not exist.

*Examples marked red are not part of the Fall 2015 syllabus.
8

1
1 1
+
=
10 10 5

Solution: a.
the closest to

Assume

c > 0.

Then there is a positive integer

among all real numbers dierent from

is a positive integer. (Why?) Let

0 < x c < x =/

1
n

= c

1
> 0.
m

Then for any

for any positive integer

1/m

such that

and of the form

> 0,

1/n

is

where

we have

f (x) = 0 f (x) 0 = 0 0 = 0 < .


Therefore

Assume

lim f (x) = 0.
xc

c < 0.

Take

= c.

Then for any

> 0,

we have

1
for any positive integer n
n
f (x) = 0 f (x) 0 = 0 0 = 0 < .

0 < x c < x < 0 x =/

Therefore

lim f (x) = 0.

b.

be a real number and assume that

Let

xc

there exists a

>0

such that for all

lim f (x) = L.

Then for every

> 0,

L = 0 L = f (x) L < L/2

is not

x0

x,

0 < x 0 < f (x) L < .


If

is not 0, let

= L/2 > 0.

Then there is a

>0

such that

0 < x < f (x) L < L/2 .


Take

x = /2.

Then

0 < x <

is true, but

true. We have a contradiction.


On the other hand, if

L = 0,

let

= 1/2.

Then there is a

>0

such that

0 < x < f (x) < 1/2 .


If

but

is a positive integer satisfying

1 = 1 = f (x) < 1/2

Hence

11.

lim f (x)
x0

n > 1/ ,

take

x = 1/n.

Then

0 < x <

is true,

is not true. Again we have a contradiction.

cannot exist.

Show that the equation

x2 10 = x sin x

has a real solution.

Solution:

f (10) =

Consider the function f (x) = x2 10 x sin x. Then f (0) = 10 < 0 and


2
10 10 10 sin(10) = 90 10 sin(10) 90 10 = 80 > 0. Note that f is

continuous on
the function

in

( 0, 10)

[ 0, 10].

Therefore we can apply the Intermediate Value Theorem to

[ 0, 10] for the value 0 to conclude that there is a point


f (c) = 0. This c is also a solution of the given equation.

on the interval

such that

12.

Consider the equation

x2
= cos x .
4

a.

Show that this equation has at least one real solution.

b.

Show that this equation has at least two real solutions.

c.

Show that this equation has at least three real solutions.

Solution:

Let

f (x) = 1

correspond to the zeros of


As

f (0) = 0, x = 0

Now observe that

x2
cos x.
4
f.

The solutions of the equation

x2
= cos x
4

is one zero.

f (/2) = 1 2 /16 > 0

f () = 2 2 /4 < 0

and

as

4 > > 3.

As

is continuous on the entire real line, applying the Intermediate Value Theorem to
the function

[/2, ]

on the interval

interval such that

f (c) = 0.

Finally, as the function

we conclude that there is a point

in this

This is our second zero.

is even, we have

f (c) = f (c) = 0,

and

x = c

is our third

zero.

13.

Show that at any moment there are two antipodal points on the equator of the Earth with

the same temperature.

Solution:

First we will make a mathematical model of the problem.

We will

. We
in radians,
= 7/4, and

consider the equator as a circle, and use the longitude as our coordinate
choose the positive direction for
and let it take any real value.

= 15/4,

among other values.

the point corresponding to

T ( + 2) = T ()

for all

want to show that there is a

to correspond to the East, measure


So 45 W corresponds to = /4,
Note that

We let

T ()

T is a continuous
T (c + ) = T (c).

We will assume that

such that

corresponds to the antipode of

denote the temperature at

We have

function. We

f () = T ( + ) T (). Note that since T is continuous, f is


continuous. Our quest to nd a c such that T (c + ) = T (c) is equivalent to nd a
c such that f (c) = 0. If f (0) = 0, then we let c = 0 and we are done. Suppose that
f (0) =/ 0. Observe that f (0) = T () T (0) = T () T (2) = f (). In other words,
f (0) and f () have opposite signs. Now we apply the Intermediate Value Theorem
to f on the interval [0, ] for the value 0, and conclude that there is a point c in
[0, ] such that f (c) = 0. We are done.
Consider the function

Remark:

It is possible to show that at any moment there are two antipodal points on Earth

with the same temperature and the same pressure.

10

14.

Find all tangent lines to the graph of

y = x3

that pass through the point

(2, 4).

Solution:

As dy/dx = d(x3 )/dx = 3x2 , the equation of the tangent line through
3
3
2
a point (x0 , x0 ) on the graph is y x0 = 3x0 (x x0 ). This line passes through
(2, 4) exactly when 4 x30 = 3x20 (2 x0 ), or in other words, x30 3x20 + 2 = 0. We

x0 = 1 is a root of this polynomial. Therefore we have the factorization

(x0 1)(x20 2x0 2). The roots of the quadratic factor are x0 = 1 3.

Therefore the tangent lines to y = x3 at the points (1, 1), (1 +


3,
10
+
6
3), and

(1 3, 10 6 3) pass through
(2,
4)
. The equations of these lines are y = 3x 2,

y = (12 + 6 3)x (20 + 12 3), and y = (12 6 3)x (20 12 3), respectively.
observe that
x30 3x20 + 2 =

15.

Evaluate the limit

Solution:

lim
x0

lim
x0

1 + sin2 x2 cos3 x2
.
x3 tan x

1 + sin2 x2 1 1 cos3 x2 x
1 + sin2 x2 cos3 x2
+

= lim
x0
x3 tan x
x4
x4
tan x
sin x2 2
1
x
1 cos3 x2
= lim ((( 2 )
)
+
)
4
2
x0
x
x
tan x
1 + sin x2 + 1

Now we observe that:

sin x2
=1
x0 x2
1
1
1
lim
=
=
2 2
2 2
x0
1 + sin x + 1
1 + sin 0 + 1 2
x
lim
=1
x0 tan x
lim

11

and

1 cos3 x2
1 cos x2
=
lim
(
(1 + cos x2 + cos2 x2 ))
x0
x0
x4
x4
2 sin2 (x2 /2)
(1 + cos x2 + cos2 x2 ))
= lim (
x0
x4

lim

1
sin(x2 /2)
= (lim
) lim(1 + cos x2 + cos2 x2 )
x0
2 x0 x2 /2
1
= 12 3
2
3
= .
2

1 + sin2 x2 cos3 x2
1 3
lim
= (1 + ) 1 = 2 .
3
x0
x tan x
2 2

Therefore:

16.

Find the equation of the tangent line to the graph of

x = 1.

y = sin2 (x3 /6)

at the point with

dy
= 2 sin(x3 /6) cos(x3 /6) 3x2 /6 .
dx
3
dy
= 2 sin(/6) cos(/6) /2 =
.
dx x=1
4

Solution:

Since

y = sin2 (x3 /6)

yx=1 = 1/4,

using the point-slope formula we nd the equation of the tangent

line as

1
y =
4

3
(x 1)
4

or, after some reorganization,

y=

17.

Let

3
1 3
x+
.
4
4

2x + x2 sin( )

f (x) =

a.

Find

b.

Show that

f (x)

for all

Therefore,

x.

is not continuous at

0.

12

if

x =/ 0,

if

x = 0.

Solution: a.

For

f (x) =
for

x =/ 0.

For

x=0

x =/ 0 we compute the derivative using the rules of dierentiation:

d
(2x + x2 sin(1/x)) = 2 + 2x sin(1/x) + x2 cos(1/x) (1/x2 )
dx

we must use the denition of the derivative:

f (0 + h) f (0)
2h + h2 sin(1/h)
= lim
h0
h0
h
h
1
= lim 2 + lim h sin( ) = 2 + 0 = 2
h0
h0
h

f (0) = lim

lim h sin(1/h) = 0

whose proof uses the Sandwich (or


h0
Squeeze) Theorem. Here is a recap of the proof: Since sin(1/h) 1 for all h =
/0
Here we used the fact that

we have

h sin(1/h) = h sin(1/h) h

for all

1
h h sin( ) h
h
As

lim h = 0 = lim(h),
h0

lim h sin(1/h) =
h0

h0
0.

To summarize:

b.

it

follows

h =/ 0.

for all

by

the

1
1

2 + 2x sin( ) cos( )

x
x

f (x) =

lim f (x).

h =/ 0 .
Sandwich

if

x =/ 0,

if

x = 0.

Theorem

that

(a)

and lim 2x sin(1/x) = 0 as in part


.
x0
x0

However lim cos(1/x) does not exist. It follows that lim f (x) does not exist and
x0
x0
hence f is not continuous at 0.
Consider

x0

We have

lim 2 = 2

Therefore

13

18.

Find

d2 y

if y is a dierentiable function of x satisfying the equation x3 +2y 3 = 5xy .


dx2 (x,y)=(2,1)

Solution:
x3 + 2y 3 = 5xy

d/dx

3x2 + 6y 2

dy
dy
= 5y + 5x
dx
dx

()

x = 2, y = 1

12 + 6

dy
dy
= 5 + 10
dx
dx

dy
=7
dx
dy 7
=
dx 4

at

(x, y) = (2, 1)

Now we dierentiate the equation marked

14

()

with respect to

to nd the second

derivative.

3x2 + 6y 2

dy
dy
= 5y + 5x
dx
dx
d/dx

dy
d2 y
dy
dy
d2 y
6x + 12y ( ) + 6y 2 2 = 5
+5
+ 5x 2
dx
dx
dx
dx
dx

x = 2, y = 1,

dy 7
=
dx 4

d2 y
7
d2 y
7
12 + 12 ( ) + 6 2 = 10 + 10 2
4
dx
4
dx

d2 y 125
=
dx2
16

Remark:

An alternative approach is to solve

dierentiate this with respect to

19.

at

from

(x, y) = (2, 1)

(),

viz.

y =

y .

to nd

5y 3x2
,
6y 2 5x

and then

A piston is connected by a rod of length 14 cm to a crankshaft at a point 5 cm away from

the axis of rotation of the crankshaft. Determine how fast the crankshaft is rotating when the
piston is 11 cm away from the axis of rotation and is moving toward it at a speed of 1200
cm/sec.

Solution:

Let

P (x, y)

and

Q(a, 0)

be the ends of the connecting rod as shown in

the picture. The axis of rotation of the crankshaft passes through the origin of the

xy -plane

and is perpendicular to it.

The point

where the rod is connected to

crankshaft moves on a circle with radius 5 cm and center at the origin. The point

Q where the rod is connected to the piston moves along the positive x-axis.
angle between the ray OP and the positive x-axis.
y
P (x, y)

14 c
m

Q(a, 0)
x

x2 + y 2 = 25

The question is:

a = 11 cm

and

da
d
= 1200 cm/sec
=?
dt
dt
15

is the

We have

x2 + y 2 = 52

(I)

and

(x a)2 + y 2 = 142 .

(II)

At the moment in question a = 11 cm. Substituting this in (I) and (II) we obtain
x2 + y 2 = 52 and (x 11)2 + y 2 = 142 . Subtracting the second equation from the
rst
2
2
2
gives 22x11 = 5 14 , and solving for x we get x = 25/11 cm. Then y = 20 6/11
cm.
Dierentiating (I) and (II) with respect to time

x
and

(x a) (
At the moment in question

y = 20 6/11

a = 11

we obtain

dx
dy
+y
=0
dt
dt

(III)

dx da
dy
)+y
=0.
dt dt
dt

(IV)

cm,

da
= v = 1200
dt

cm/sec,

x = 25/11

cm and

cm. Substituting these in (III) and (IV) we get

dy
dx
+4 6
=0
dt
dt

(V )

and

dy
dx
+ 20 6
= 146v .
(VI)
dt
dt
dx
dx 146
Subtracting 5 times (V) from (VI) we nd 121
= 146 v , and hence
=
v.
dt
dt 121
dy
365
v.
Substituting this back in (V) gives
=
dt 242 6
146

Now we are ready to compute

d
.
dt

Since

tan = y/x,

dierentiation gives

dy
dx
x y
d
sec2
= dt 2 dt
dt
x
and using

sec2 = 1 + tan2 = 1 + (y/x)2

we obtain

dy
dx
d x dt y dt
.
=
dt
x2 + y 2

25
20 6
dx 146
Plugging x =
cm, y =
cm,
=
v,
11
11
dt 121

d
73
1460 6
v=
gives
=
radian/sec.
dt
11
110 6

16

and

dy
365
v
=
dt 242 6

in this formula

Remark:
Remark:

2 5 x cos

1460 6
11

radian/sec is

1460 6 60

11
2

rpm or approximately 3105 rpm.

This problem has a shorter solution if we use the law of cosines. Start with
= 142 and dierentiate with respect to t to obtain

x2 + 52

d 5 cos 11
=
v.
dt
5 sin
Put

x = 11

cm in the rst equation to nd

cos = 5/11

and then

sin = 4 6/11.

Now

substituting these in the second equation gives the answer.

20.

Determine how fast the length of an edge of a cube is changing at the moment when the
length of the edge is 5 cm and the volume of the cube is decreasing at a rate of 100 cm3/sec.

Solution:

a denote the length of an edge of the cube, and V denote the volume
of the cube. Then we have V = a3 . Dierentiating with respect to time t gives
dV
da
dV
= 3a2
. Substituting
= 100 cm3/sec and a = 5 cm for the moment in
dt
dt
dt
da
4
question, we obtain
= cm/sec. Therefore the length of the edge is decreasing
dt
3
4
at a rate of
cm/sec at that moment.
3

21.

Let

We measure the radius and the height of a cone with 1% and 2% errors, respectively. We

use these data to compute the volume of the cone. Estimate the percentage error in volume.

Solution:

r , h,

Let

and

be the radius, the height and the volume of the cone,

respectively.

V =

2
2

dV
dr dh
r h dV =
rhdr + r2 dh
=2
+
.
3
3
3
V
r
h

Since the error in

is 1% we have

dr
1% .
r

Similarly

dh
2% .
h

triangle inequality we obtain

dV
dr dh
dr
dh
= 2
+ 2 + 2 1% + 2% = 4% .
V
r
h
r
h

The error in volume is 4% .

17

Now using the

22.

A cone of radius 2 cm and height 5 cm is lowered point rst into a tall cylinder of radius

7 cm that is partially lled with water. Determine how fast the depth of the water is changing
at the moment when the cone is completely submerged if the cone is moving with a speed of 3
cm/s at that moment.

Solution:

Let

and

be the radius and the height of the part of the cone that is

under the water level, respectively. Let


and let

be the depth of the water in the cylinder

be the vertical distance from the tip of the cone to the bottom of the

cylinder. Let

V0

be the volume of the water.

Then

2
4
V0 = 7 L r2 h = 72 L ( h) h = 49L h3
3
3 5
75
2

where we used the fact that

r/h = 2/5.

Now dierentiating this with respect to time

0=

gives

d
dL 4 2 dh
V0 = 49
h
.
dt
dt 25 dt

In particular at the moment when the cone is completely submerged we have


cm and

49

h=5

dL
dh
=4
.
dt
dt

On the other hand, at the same moment

h = L y
because

dy/dt = 3

dh dL
dh dL dy
=

=
+3
dt dt dt
dt dt

cm/s as the cone is being lowered at a speed of 3 cm/s.

From these two equations we obtain

dL/dt = 4/15

cm/s. In other words, the depth

of the water is increasing at a rate of 4/15 cm/s at that moment.

18

23.

A water tank has the shape of an upside-down cone with radius 2 m and height 5 m. The

water is running out of the tank through a small hole at the bottom. Assume that the speed of
the water owing through the hole is proportional to the square root of the depth of the water
in the tank.

a.

In this part, suppose that the water is running out at a rate of

of the water in the tank is

m.

3 m3/min when the depth

Find the rate at which the water level is changing at this

moment.

b.

In this part, suppose that the water level is falling at a rate of

1/3 m/min when the tank

is full. Find the rate at which the water level is changing when the depth of the water in the
tank is

c.
5

m.

In this part, suppose that it takes 3 minutes for the depth of the water to decrease from

m to

m. Find how long it takes for the full tank to completely drain.

Solution:

Let

water, and let

r
V

and

denote the radius and the height of the cone formed by the

denote the volume of the water. Using the fact that

obtain

we

V =
and hence:

In part

r/h = 2/5,

2
2
4 3
r h = ( h) h =
h
3
3 5
75
dV 4 2 dh
=
h
dt 25
dt

(a), we are given that dV /dt = 3 m3 /min when h = 4 m.

dh/dt we obtain dh/dt = 75/(64)


rate of 75/(64) m/min in part (a).

in the equation above and solving for


Hence the water level is falling at a

Substituting these
m/min.

Now we will use the condition that the speed of the water owing through the hole
is proportional to the square root of the depth of the water in the tank. This means

dV
= k h
dt
for some positive constant

k.

Substituting this in the previous equation we obtain:

4 2 dh
k h =
h
25
dt
19

()

In part
these in

(b), we are given


() we obtain:

On the other hand,

that

dh/dt = 1/3

m/min when

h=5

m.

Substituting

4 2
1
k 5 =
5 ( )
25
3

() gives

4 2 dh
k 4 =
4
25
dt

h = 4 m.
5 5/24 m/min.
when

(b).

For part

dh/dt =

Hence the water level is falling at a rate of 5 5/24 m/min in part

Now solving for

dh/dt

from these two equations gives

(c), we rewrite () in the form


4 3/2
h dh = k dt
25

and integrate to obtain

8 5/2
h = kt + C
125
for some constant C . In other words, h5/2 = at + b for some constants a and b. As
h = 5 m when t = 0 min, we have b = 55/2 . Using the condition in part (c) that h = 4
m when t = 3 min we obtain 45/2 = a 3 + 55/2 , so a = (55/2 45/2 )/3. Finally, the
tank is empty when h = 0 and this happens when t = b/a = 3 55/2 /(55/2 45/2 ) min.
Hence the tank drains in 3/(1 (4/5)5/2 ) minutes in part (c).

20

24.

Find the absolute maximum value and the absolute minimum value of

on the interval

[1, 6].

Remark: How to nd the absolute maximum and the absolute


continuous function f on a closed interval [ a, b] of nite length:
i. Compute

f (x) = x4/3 x x1/3

minimum values of a

f .

ii. Find the critical points of


iii. Add the endpoints

and

iv. Compute the value of

f
b

in

(a, b).

to this list.

at each point in the list.

v. The largest value is the the absolute maximum and the smallest value is the absolute
minimum of

Solution:
x=0

on

[a, b].

4
1
f (x) = x1/3 1 x2/3 .
3
3

The derivative is not dened at

is a critical point. Next we solve f (x)

x = 0, therefore

= 0.

4 1/3
1
x 1 x2/3 = 0 we let z = x1/3 to obtain the equation 4z 3 3z 2
3
3
1 = 0. Since z = 1 is a root, we have the factorization 4z 3 3z 2 1 = (z 1)(4z 2 +z +1).
As the quadratic factor has no real roots, z = 1 is the only solution. Therefore
x = z 3 = 13 = 1, which belongs to the interval [1, 6], is the only other critical point.

In the equation

We have f (0) = 0, f (1) = 1, f (1) = 3, and f (6) = 5


5 61/3 6 > 3 5 61/3 > 9 53 6 > 93 725 > 721.

61/3 6.

We conclude that the absolute maximum and minimum values of


on the interval [1, 6] are 5 61/3 6 and 1, respectively.

25.

f (x) = x4/3 xx1/3

Find the absolute maximum and the absolute minimum values of

interval

f (x) =

[ 0, ).

Remark:

Observe that

x2

x+1
+x+9

on the

When looking for the absolute maximum and the absolute minimum values of a

continuous function on an interval that is not necessarily closed or of nite length, a modied
version of the algorithm above can be used.

In Step iv , if an endpoint does not belong to the interval, then we compute the appropriate
one-sided limit of the function at that point instead of the value of the function.
In Step v , if the largest value (which can be

occurs only as a limit, then we conclude

that there is no absolute maximum. Similarly for the smallest value.

Solution:
and

x = 2.

f (0) =

We compute
Only

x=2

1
1
, f (2) =
,
9
5

is in the

and

x2 + 2x 8
. The roots of f (x) = 0 are x = 4
(x2 + x + 9)2
interval [ 0, ). So our list is 0, 2, and .

f (x) =

lim f (x) = 0.

x
and there is no absolute minimum.

Since

21

1 1
> > 0,
5 9

the absolute maximum is

1
5

26.

Suppose that a function

f (x0 ) = 0

for some

Solution:

x0 > 0.

x0 > 0,

Since

f (x) = f (x/2) for all x and f (0) = 1.


x1 such that 0 < x1 < x0 and f (x1 ) = 0.

satises

then there is

is dierentiable,

is continuous. Suppose that

Show that if

f (x0 ) = 0 for some

f on [ 0, x0 ] to conclude that there


f
(x
)

f
(0)
1
0
is c such that 0 < c < x0 and f (c) =
= < 0. Then f (x) = f (x/2)
x0 0
x0
gives f (c/2) = f (c) < 0. We also have f (0) = 1 > 0. We apply the Intermediate
Value Theorem to f on [ 0, c/2] to conclude that there is x1 such that 0 < x1 < c/2
and f (x1 ) = 0. As 0 < x1 < c/2 < c < x0 we are done.

Remark:

We apply the Mean Value Theorem to

With a little bit more work it can be shown that

f (x) > 1

27.

Show that if f is a twice-dierentiable function such that f (0)


f (1) = 5, and f (x) 0 for all x, then f (x) 1/3 for all 0 x 1.

for all

x > 0.

= 1, f (0) = 1, f (1) = 2,

Solution:

Let x1 < x2 be in [0, 1]. The Mean Value Theorem applied to the

function f on the interval [x1 , x2 ] says that there is a point c in (x1 , x2 ) such that
f (x2 ) f (x1 )
= f (c). As f (c) 0 we have f (x1 ) f (x2 ). In particular, we
x2 x1
have 1 = f (0) f (x) f (1) = 5 for all x in (0, 1).

0 < x < 1.

Let

interval

[0, x], there exists a point c1

f (c1 ) 1,

f on the
f (x) f (0)
= f (c1 ). Since
x0

By the Mean Value Theorem applied to the function


in

(0, x) such that

it follows that

f (x) x + 1

( Q)

0 < x < 1. Similarly, applying the Mean Value Theorem to the function
f on the interval [x, 1], we see that there exists a point c2 in (x, 1) such that
f (1) f (x)
= f (c2 ). Now using the fact that f (c2 ) 5 we conclude that
1x
for

f (x) 5x 3
for

0 < x < 1.

5
[0, 1].

Adding
in

(QQ)

Remark:

times the inequality

(Q) to the inequality (QQ) we get f (x) 1/3 for all x

In fact, it can be shown that

f (x) > 1/3

22

for all

in

[0, 1].

Sketch the graph of

10 1/3 10 2/3 10 1/3


x
x =
x (1 x).
3
3
3
(, 0) and (1, ).

Solution:
y

<0

on

y = 5x2/3 2x5/3 .

y =

20
10 4/3 20 1/3
x
x
= x4/3 (x + 1/2).
9
9
9
y < 0 on (1/2, 0) and (0, ).

y =

Therefore

Therefore

y > 0

y > 0

on

(0, 1),

and

(, 1/2),

and

on

These give us the following table of signs and shapes.

x
y
y

1/2

+
"

28.

^
^
^
^

inf.pt.
We compute the
minimum,

y -coordinates

"

loc.min.

loc.max.

of the important points to get

(0, 0)

for the local

(1, 3)for the local maximum,


and (1/2, 3 2) for the inection point.

3
2 > 1, we have 3 3 2 > 3. Also note that the function is continuous

Note that since


at

x = 0,

but

lim y = lim+ (

x0+

x0

and

lim y = lim (

x0
Therefore

(0, 0)

or

x = 5/2.

10 1/3
x (1 x)) = .
3

is a cusp.

Finally we nd the

x0

10 1/3
x (1 x)) =
3

x-intercepts: y = 0 5x2/3 2x5/3 = 0 2x2/3 (5 2x) = 0 x =

(The

graph is on the next page .)

23

Now we use these data to draw the graph:

Remark:

Here is the same graph drawn by Maple :

24

29.

Two corridors meet at a corner. One of the corridors is 2 m wide and the other one is 3

m wide. What is the length of the longest ladder that can be carried horizontally around this
corner?

Solution:
of

Length of the longest ladder will be equal to the absolute minimum value

in the picture.

y
2m

L
3m

x
Using the relation

3/x = y/2

we obtain

L = (x2 + 9)1/2 + (4 + y 2 )1/2 = (x2 + 9)1/2 + (4 + (6/x)2 )1/2 .


Therefore we have to minimize

2
L = (x2 + 9)1/2 (1 + )
x

for

0<x< .

We rst look at the critical points:

dL
2
2
= (x2 + 9)1/2 x (1 + ) + (x2 + 9)1/2 ( 2 ) = 0
dx
x
x
18
x + 2 (2 + 2 ) = 0 x = 181/3 m L = (22/3 + 32/3 )3/2
x
Since

lim L = and lim L = , the value at the critical point is indeed the absolute

x0+
minimum.

Hence the length of the longest ladder that can be carried around this corner is
(22/3 + 32/3 )3/2 m.

Remark:

(22/3 + 32/3 )3/2

m is approximately

7.02

25

m.

30.

Find the maximum possible total surface area of a cylinder inscribed in a hemisphere of

radius 1.

Solution:
h=

Let S be the total surface area of the cylinder,


2
1/2
(1 r ) . Hence we want to maximize

S = 2r2 + 2r(1 r2 )1/2

for

S = 2r2 + 2rh.

We have

0 r 1.

1 dS
= 2r + (1 r2 )1/2 r2 (1 r2 )1/2 = 0
2 dr
2r(1 r2
)1/2 = 2r2 1 4r2 (1 r2 ) = (2r2 1)2 8r4 8r2 + 1 = 0
8 32
1
r2 =
. Note that 2r(1 r 2 )1/2 = 2r 2 1 implies r 2
, and therefore
16
2

2+ 2
2+ 2
r2 =
and r =
.
4
2

2+ 2
We get S = (1 + 2) at the critical point r =
, S = 0 at the endpoint r = 0,
2

2 > 1 we have 1 + 2 > 2, and the absolute


and S = 2 at the endpoint r = 1. As
First we nd the critical points:

maximum value occurs at the critical point.


The maximum possible surface area of the cylinder is

26

(1 +

2) .

31.

A fold is formed on a 20 cm

30 cm rectangular sheet of paper running from the short

side to the long side by placing a corner over the long side. Find the minimum possible length
of the fold.

Solution:

ABCD be the sheet of paper and let P be the point on the edge
C is folded over. The fold runs from Q on the edge BC to R
on the edge CD . Let S be the projection of R on to the edge AB . Let L be the
length of the fold QR and let x = CQ.
AB

Let

where the corner

Q
x
A

P Q/P
B = RP /RS by the similarity
of the triangles P BQ and RSP . Hence

2
2
RP = 20x/ x (20 x) = 20x/ 40x 400 and L2 = RQ2 = RP 2 + P Q2 =
400x2 /(40x 400) + x2 = x3 /(x 10). The largest possible value of x is 20
cm. The smallest possible value of x occurs when RP = 30 cm; that is when

20x/ 40x 400 = 30 or x = 45 15 5 cm.


Then

Therefore we want to minimize


Dierentiating

where

L2 = x3 /(x 10)

2LdL/dx = 3x2 /(x 10) x3 /(x 10)2

x = 15 cm
L = 15 3 cm.
obtain

for

45 15 5 x 20.
dL/dx = 0 we
x = 15 cm we have

and setting

as the only critical point in the domain. For

cm we have

x =20 cm we have L = 20 2 cm and at the endpoint x = 45 15 5

L = 15 18 6 5 cm.

15 3

cm is the smallest of these values, we conclude that this is the smallest

At the endpoint

Since

possible value for the length of the fold.

w 2 2/3, then the length


of the shortest possible fold is 3 3w/4. Therefore, for a A4 size paper with w = 210 mm and
= 297 mm, the shortest fold is approximately 273 mm; and for a letter size paper with w = 8.5
in and = 11 in, the shortest fold is approximately 11.04 in.

Remark:

If the question is posed for a

sheet of paper with

27

32.

The Rubber Duck is a sculpture designed by Florentijn Hofman and constructed from

PVC. For the purposes of this question, we consider the Rubber Duck to consist of a spherical
head of radius

and a spherical body of radius

b.

The research shows that the cuteness

the Rubber Duck is given by

a
a

b (1 b )(a + b)
K =

if

0 a < b,

if

0 < b a.

Find the dimensions of the cutest Rubber Duck with a total surface area of

Solution:
4b2 =

The total surface area of the Rubber Duck is


400 m2 , giving b2 = 100 a2 . Therefore,

a(100 2a2 )

100 a2
K =

if

4a2 + 4b2 .

400 m2 .

Hence

4a2 +

0 a 5 2,

otherwise,

and we have to nd where the absolute maximum value of

K =
on the interval

0a5 2

100a 2a3
100 a2

occurs.

We rst nd the critical points. As

dK
(100 6a2 )(100 a2 ) (100a 2a3 )(2a)
,
=
da
(100 a2 )2
dK /da = 0

gives

a4 250a2 + 5000 = 0. Now using the quadratic

250 2502 4 5000


2
a =
= 125 25 17 ,
2
28

formula we obtain

of

which gives us four solutions





a = 5 5 + 17, 5 5 17, 5 5 + 17, 5 5 17 .

5
2. On the

a = 5 5 17

Only the rst two of these are positive and the rst one is greater than
other hand,

0 < 5 5 17 < 5 2

as

3<

17 < 5.

We conclude that

is the only critical point.


We have

K = 10 5
for


a = 5 5 17.

As

K =0

17 3
17
>0
17 1

at the endpoints

a=0

and

a=5 2

of the interval,

this is the absolute maximum value.


Therefore the cutest Rubber Duck has

33.


a = 5 5 17

m and

b=5
17 1

m.

A snowman is an anthropomorphic sculpture made from snow as well as some pieces of

coal, a carrot, a hat and a scarf. For the purposes of this question, we consider a snowman to
consist of a spherical head of radius

and a spherical body of radius

b,

and we also assume

that the snow does not melt and its density does not change while it is being sculpted.
The research shows that the cuteness

of a snowman is given by

a
a

2
2

( b ) (1 b ) (a + ab + b )
K =

if

0 a < b,

if

0 < b a.

Find the dimensions of the cutest snowman that can be built with

Solution:

The total volume of the snowman is

29

4 3 4 3
a +
b.
3
3

4/3 m3

Hence

of snow.

4 3 4 3
a +
b =
3
3

4
3

m3 , giving

b3 = 1 a 3 .

Therefore,

a2 (1 2a3 )

3
K = 1a

1
0a
,
3
2

if

otherwise,

and we have to nd where the absolute maximum value of

K =
on the interval

1
0a
3
2

a2 2a5
1 a3

occurs.

We rst nd the critical points. As

dK
(2a 10a4 )(1 a3 ) (a2 2a5 )(3a2 )
=
,
da
(1 a3 )2
dK /da = 0

4a7 9a4 + 2a =0. As we have4a7 9a4 + 2a = a(a3 2)(4a3 1),


3
2 and
a = 1/ 3 4. Hence the only critical point in
the critical points are a = 0, a =
3
the interval we are interested in is a = 1/ 4.

3
3
We have K = 1/(3 2) > 0 for a = 1/ 4. As K = 0 at the endpoints a = 0 and
1
a=
of the interval, this is the absolute maximum value.
3
2

3
3
Therefore the cutest snowman has a = 1/ 4 m and b =
3/4 m.

34.

gives

A dessert in the shape of a hemisphere with radius 1 dm is made by baking a cylindrical

cake of height

h,

and topping it with a spherical cap of ice cream and surrounding it with a
hemispherical ring of chocolate mousse as shown in the gure. If the cake costs 8/ /dm3 , the
ice cream costs 9/ /dm3 and the chocolate mousse costs 12/ /dm3 , determine the value of

for

(a) the least expensive and (b) the most expensive dessert that can be made.

You may use the fact that the volume of a hemispherical ring of height

h is 2h3 /3.
Ice cream
(9/

/dm3 )

Cake
(8/

/dm3 )

Chocolate
mousse

30

(12/

/dm3 )

Solution:

We have

Cost

12
(Volume

of the ring) +

8
(Volume

of the cylinder)

9
(Volume of the cap)

9 2
2
12 2 3 8
h + (1 h2 )h + ( 13 h3 (1 h2 )h)
=
3

3
3
+

= 6 h + 3h3
and hence we want to nd the absolute maximum and the absolute minimum values
of
Cost

= 6 h + 3h3

We rst nd the critical points. As


gives

h = 1/3

dm and

h = 1/3 dm.
52/9

and at that point the cost is

for

d(Cost)/dh = 1 + 9h2 , setting this equal to zero


Only the rst of these are in the interval [0, 1],
.

Next we look at the endpoints of the interval.


dm gives Cost

=8

35.

h=1

h=0

dm gives Cost

=6

, and

h=1

Therefore the least expensive dessert has


has

0h1.

h = 1/3 dm, and the most expensive dessert

dm.

We want to build a greenhouse that has a half cylinder roof of radius

and height

mounted horizontally on top of four rectangular walls of height h as shown in the gure. We
have 200 m2 of plastic sheet to be used in the construction of this structure. Find the value
of

for the greenhouse with the largest possible volume we can build.

Solution:

We have

200 m2 = Total

Surface Area

1
1
= 2 (r + 2r)h + 2 r2 + 2r r
2
2


walls
half-disks

31

top

and hence

h=

100
(
r).
3 r

the greenhouse. Then

0 < r 10.

In particular,

Let

denote the volume of

1
V = r 2r h + r2 r
2
bottom

top

and substituting

in terms of

Maximize

in this, our problem becomes:

V =

(400r r3 )
6

We rst nd the critical points in the interval

for

0 < r 10.

(0, 10].

20
V = (400 3r2 ) = 0 r = m
dr
6
3

Since neither of these satises 0 < r 10 as


3 < 2, there are no critical points
our interval. Now we look at the endpoints. At r = 0 we have lim V = 0, and
r0+
r = 10 we have V = 500 .
Therefore the maximum possible volume is

h=0

36.

Let

500

m3 and occurs when

m and

m.

be a continuous function.

x2

a.

Find

f (4)

if

b.

Find

f (4)

if

Solution:

a.

f (t) dt = x sin x

f (x)

t2 dt = x sin x
x2

x.

for all

for all

x.
FTC1

x
d
d

f (t) dt =
(x sin x)

dx 0
dx
x = 2 we get f (4) = /2.

f (t) dt = x sin x

0
f (x ) 2x = sin x + x cos x.
2

Now letting

f (x)

t3
f (x)3
b.
t dt = x sin x ] = x sin x
= x sin x.
3 0
3
0
gives f (4) = 0.
f (x)

Remark:

r = 10

in
at

One might ask if such functions exist. In part

(b), f (x) =

Hence

3x sin x

x = 4

is the unique

function satisfying the given condition.


In part

(a),

anything for

sin x

f (x) =
+ cos x for x > 0,
2
2 x
x < 0 so long as it is continuous.

32

and

f (0) =

by continuity; but it can be

37.

Compute

d2 y

dx2 (x,y)=(0,0)

if

x+y

Solution:

satisfying the equation:

we obtain:

is a dierentiable function of

et dt = xy
2

Dierentiating the equation with respect to

x+y

et dt = xy
2

d/dx

x+y
d
d
2
et dt =
(xy)

dx 0
dx

FTC1

e(x+y)
2

d
d
(x + y) =
(xy)
dx
dx

e(x+y) (1 +
2

dy
dy
)=y+x
dx
dx

(e)

x = 0, y = 0

dy
= 1
dx
Now dierentiating (e) with respect to

at

(x, y) = (0, 0)

again we get:

e(x+y) (1 +
2

dy
dy
)=y+x
dx
dx

d/dx

d
dy
d
dy
2
(e(x+y) (1 + )) =
(y + x )
dx
dx
dx
dx

(x+y)2

2
dy
dy dy
d2 y
2 d y
(2(x + y))(1 + ) + e(x+y)
=
+
+
x
dx
dx2 dx dx
dx2

x = 0, y = 0, dy/dx = 1

d2 y
= 2
dx2

33

at

(x, y) = (0, 0)

38.

Suppose that

is a continuous function satisfying

f (x) = x
for all

x,

and

f (t) dt + x3

f (c) = 1.

is a real number such that

Solution:

Express

f (c)

in terms of

only.

We have:
FTC1

x
x
x
d
d

f (x) = f (t) dt + x f (t) dt + 3x3 = f (t) dt + xf (x) + 3x2


dx
dx 0
0
0
Now substituting

x = c and using the facts that f (c) = c

c
0

f (t) dt + c3

and

f (c) = 1

we obtain:

f (c) =

Remark:

c
0

f (t) dt + cf (c) + 3c2 =

It can be shown that

f (x) = 2x(ex

2 /2

f (c) c3
1
+ c + 3c2 = + c + 2c2
c
c

1) is the only function that satises the given

condition.

39.

Evaluate the limit

Solution:

3
sin(xt ) dt
lim 0
x0
x5

We rst make the change of variable

1/3

0 sin(xt ) dt = x
3

t = x1/3 u, dt = x1/3 du,

x4/3

sin(u3 ) du

Now we have:

x4/3

3
x1/3 0 sin(u3 ) du
0 sin(xt ) dt
lim
= lim
x0
x0
x5
x5
x4/3
sin(u3 ) du

= lim 0
x0
x16/3
d x4/3
3
0 sin(u ) du

dx
= lim
x0
16/3 x13/3
L'H

sin(x4 ) 4/3 x1/3


x0
16/3 x13/3
sin(x4 ) 4/3 x1/3
= lim
x0
16/3 x13/3
1
sin(x4 )
= lim
4 x0 x4
1
1
= 1=
4
4

FTC1

= lim

34

to obtain:

40.

Suppose that f is a twice-dierentiable function satisfying f (0) =


8, f (2) = 5, f (2) = 3, f (2) = 7 ; and also suppose that the function

g(x) =
has a critical point at

x = 2.

2, f (0) = 11, f (0) =

1 x
f (t) dt
x 0

Determine whether the critical point of

at

x=2

is a local

minimum, a local maximum or neither.

Solution:

We rst observe that

d
d 1 x
g(x) =
(
f (t) dt)
dx
dx x 0
FTC1

x
x
1
1d
f
(t)
dt
+
f (t) dt

x2 0
x dx 0
x
1
1
= 2 f (t) dt + f (x)
x 0
x

and

as

g (2) = 0

we

must

have

1/4

2
0

f (t) dt + 1/2 f (2) = 0,

and

hence

0 f (t) dt = 2f (2) = 10.


Now dierentiating a second time we obtain that:

x
d
1
1
d2
g(x)
=
(

f
(t)
dt
+
f (x))

dx2
dx
x2 0
x
FTC1

x
x
1 d
1
2
1
f
(t)
dt

f
(t)
dt

f
(x)
+
f (x)

x3 0
x2 dx 0
x2
x
x
2
1
1
1
= 3 f (t) dt 2 f (x) 2 f (x) + f (x)
x 0
x
x
x

Substituting

x=2

in this we get

g (2) =
and conclude that

41.

Suppose that

1
1 2
1
3
f (t) dt f (2) + f (2) = < 0

4 0
2
2
2

has a local maximum at

x = 2.

is a continuous and positive function on

[0, 5],

and the area between the

y = f (x) and the x-axis for 0 x 5 is 8. Let A(c) denote the area between the graph
y = f (x) and the x-axis for 0 x c, and let B(c) denote the area between the graph of
dR
= 7, nd
y = f (x) and the x-axis for c x 5. Let R(c) = A(c)/B(c). If R(3) = 1 and
dc c=3
f (3).
graph of
of

Solution:

We have

A(3) = B(3) = 4.

A(3) + B(3) = 8

and

35

R(3) = 1 A(3) = B(3),

implying

As

A(c) =

c
0

f (t) dt

dierentiating these with respect to


Calculus Part 1 we obtain A (c) =
A (3) = f (3) and B (3) = f (3).

and

B(c) =

5
c

f (t) dt ,

c and using the Fundamental Theorem of


f (c) and B (c) = f (c). In particular,

On the other hand,

d
d A(c) A (c)B(c) A(c)B (c)
R(c) =
=
dc
dc B(c)
B(c)2
and letting

7=

Let

gives

d
A (3)B(3) A(3)B (3) f (3)B(3) + A(3)f (3) f (3)
R(c) =
=
=
dc
B(3)2
B(3)2
2
c=3

and hence

42.

c=3

f (3) = 14.

be a continuous function and let

g(x) =
Express

g (x)

f (x)

in terms of

Solution:

For

1 < x < 1

g(x) =

x
1

= x

for

1
1

1 < x < 1.

we rewrite the denition of

f (t) (x t) dt +
x

f (t) x t dt .

f (t) dt

x
1

1
x

g(x)

as follows:

f (t) (t x) dt

f (t)t dt +

1
x

f (t)t dt x

1
x

f (t) dt

Now we dierentiate using the FTC1 to obtain:

g (x) =

=
=

x
1

x
1
x
1

x
x
d
d
f (t) dt
f (t)t dt

dx 1
dx 1
1
1
1
d
d
+
f (t)t dt f (t) dt x
f (t) dt

dx x
dx x
x

f (t) dt + x

f (t) dt + xf (x) f (x)x f (x)x


f (t) dt

Using FTC1 again we compute

1
x

1
x

f (t) dt + xf (x)

f (t) dt

g (x)

to nd

x
1
d
d
f (t) dt
f (t) dt

dx 1
dx x
= f (x) + f (x)
= 2f (x)

g (x) =

for

1 < x < 1.
36

43.

Evaluate the limit

lim (n (

1
1
1
+
++
)) .
2
2
(2n + 1)
(2n + 3)
(4n 1)2

1
on the interval [2, 4]. If we divide this
x2
2
2k
interval into n subintervals of equal length
using the points xk = 2 +
, 0 k n,
n
n
2k 1
and choose our sample points to be the midpoints ck = 2 +
, 1 k n, of these
n

Solution:

Consider the function

f (x) =

subintervals, the Riemann sum for these data becomes

k=1

k=1

f (ck ) xk = f (2 +

n
2k 1 2
2n
) =
n
n k=1 (2n + 2k 1)2

and the denition of the denite integral gives

4 dx
2n
1 4
1 1 1
=
=

] = + = .

2
2
x 2
4 2 4
2 x
k=1 (2n + 2k 1)
n

lim
n
Therefore

lim (n (

44.

1
1
1
1
+
++
)) = .
2
2
2
(2n + 1)
(2n + 3)
(4n 1)
8

When pirates retire, they live on the Square Island which has the shape of a square with

10 hectometer (=hm) long sides. Having lived all their lives on it, the retired pirates want to
be as far away from the sea as possible. As a result, the pirate population density

p(x)

at a

point on the Square Island is proportional to the distance x of the point from the shore and
reaches its largest value of 15 pirate/hm2 at the center of the island. Find the total number N
of the pirates on the island.

37

Solution:

p(x) = 3x. Then


x hm lie
and 10 2x hm long.

Firstly, we have

island whose distance to the shore are


to the sides of the island

we observe that the points on the


on a square whose sides are parallel

Now consider another square which consists of points whose distances to the shore
are x+x hm for some small positive x, and choose a x which lies between x and

x + x.

Then the number of pirates living in the strip between these two squares is
4 (10 2x) p(x ) x.

approximately
Hence

can be approximated by the Riemann sum

N 4 (10 2xi ) p(xi ) xi


i=1
for a partition 0 = x0 < x1 < < xn1 < xn = 5 of the interval
xi in [xi1 , xi ] for 1 i n, and in the limit we obtain:

[0, 5] and sample points

N = 4(10 2x)p(x) dx
0
This gives:

N =

5
0

5 2 1 3 5
4(10 2x)(3x) dx = 24 (5x x ) dx = 24 [ x x ] = 500
2
3 0
0
5

There are 500 pirates living on the island.

Remark:

Imagine that the entire landscape of the Square Island consists of just a mountain

and the altitude of a point that is horizontally

hm away from the shore is

p(x)

hm. Then

the island has the shape of a square pyramid with a 10 hm by 10 hm base and 15 hm height,
and

is now the volume of the island. Hence we have:

1
N = (height)(area
3

of the base)

38

1
15 102 = 500
3

Remark:
45.

Compare this example with

Example 30 in Part 2.

Evaluate the following integrals.

a.

2
2
x sin(x ) cos(x ) dx

b.

1
0 x 1 x dx

Solution: a.

Let

u = sin(x2 ).

Then

du = 2x cos(x2 ) dx,

and

1
1 u2
1
2
2
2
2
x sin(x ) cos(x ) dx = 2 u du = 2 2 + C = 4 sin (x ) + C .

b.

Let

u = 1 x.
1

Then

du = dx,

In part

x = 0 u = 1, x = 1 u = 0.

Therefore:

0
1

x 1 x dx = (1 u)u1/2 (du) = (u1/2 u3/2 ) du


1

=[

Remark:

and

u3/2
3/2

1
u5/2

5/2

] =
0

2 2 4
=
3 5 15

(a), if we let u = cos(x2 ), then we obtain the answer


1
2
2
2
2
x sin(x ) cos(x ) dx = 4 cos (x ) + C ;

39

1
sin(2x2 ), and then let u = sin(2x2 ),
2
1
2
2
2
x sin(x ) cos(x ) dx = 8 cos(2x ) + C .

and if we rst observe that


obtain the answer

sin(x2 ) cos(x2 ) =

then we

These are all correct answers. If we write

1
2
2
2
2
x sin(x ) cos(x ) dx = 4 sin (x ) + C1
1
2
2
2
2
x sin(x ) cos(x ) dx = 4 cos (x ) + C2
1
2
2
2
x sin(x ) cos(x ) dx = 8 cos(2x ) + C3
then

46.

C2 = C1 +

1
4

Show that

C3 = C1 +

1
.
8

a
a
f (x)
0 f (x) + f (a x) dx = 2

Solution:
Let

and

Let

u = a x.

for any positive continuous function on

[0, a] .

a
f (x)
I =
dx .
0 f (x) + f (a x)

Then

du = dx, x = 0 u = a, x = a u = 0,

and

a
0
a
f (x)
f (a u)
f (a x)
I =
dx =
(du) =
dx .
0 f (x) + f (a x)
a f (a u) + f (u)
0 f (x) + f (a x)
Therefore

a
f (x)
f (a x)
dx +
dx
0 f (x) + f (a x)
0 f (x) + f (a x)
a f (x) + f (a x)
dx
=
0 f (x) + f (a x)
a

2I =

=
and

I=

a
0

dx = a

a
.
2

(See

the remark on the next page .)

40

Remark:

Here is an explanation of what is going on with no integral signs: Consider the

x- and
y = f (x)/(f (x) + f (a x)) is

rectangle with a vertex at the origin, and sides along the positive

lengths

and 1, respectively.

respect to the center

The graph of

(a/2, 1/2)

a
I= .
2

Let

y -axes

and

symmetric with

is the area of the lower half, we have

y = x x2 and
R about the x-axis.

be the region bounded by the parabola

volume of the solid generated by revolving

the

x-axis,

and let

a.

Express

as an integral using the disk method. (Do not compute! )

b.

Express

as an integral using the cylindrical shell method. (Do not compute! )

Solution:

The parabola intersects the

x-axis at x = 0 and x = 1.

For part

(a).

(b), the cylindrical shell method gives a y-integral,


d

V = 2 (radius
c

of the shell)(height of the shell) dy

41

be the

Therefore we have

V = R(x)2 dx = (x x2 )2 dx
0
0
for part

with

of this rectangle, and therefore divides it into two regions of

equal area. Since the area of the rectangle is

47.

the

as the region is revolved about the

x-axis.

We have

c = 0

and

d = 1/4 ,

the

y-

coordinate of the highest point of the parabola. The radius of the shell is the vertical
distance from the red rectangle in the gure to the

x-axis,

which is

y.

The height

of the shell is the horizontal length of the rectangle; that is, the dierence between
the

x-coordinates

equation

y=

of the right and the left sides of the rectangle.

xx2 for

x, we nd these values as x =

respectively. Hence

V = 2

1/4
0

y(

1+

1+

1 4y
2

and

1 4y 1 1 4y

) dy .
2
2

42

By solving the

x=

1 4y
,
2

48.

R be the region bounded by the curve y 2 = x2 x4 . Let V be the volume obtained by


rotating R about the x-axis. Let W be the volume obtained by rotating R about the y -axis.
Let

a.

Express

b.

Express

c.

Compute

using both the disk method and the cylindrical shells method.

Solution:

and

W.

By symmetry,

is 2 times the volume obtained by revolving the portion

x-axis. In the disk method, the red


vertical rectangles, when revolved about the x-axis, form the disks that are used in

the computation of V , and therefore the radii of the disks are given by
x2 x4 .
of

using both the disk method and the cylindrical shells method.

lying in the rst quadrant about the

Hence:

V = 2 (radius
0

Again by symmetry,
of

2
of disk) dx

1
= 2 ( x2 x4 )2 dx
0

is 2 times the volume generated by revolving the portion

lying in the rst quadrant about the

y -axis.

This time in the washer method,

the green horizontal rectangles, when revolved about the


that are used in the computation of

y -axis,

form the washers

W.

To nd the outer and the inner radii of the washers we have to solve y 2 =
x2 x4 for x.
2
2
2
2
2
Applying the quadratic formula to (x ) x +y = 0 we obtain x = (1 1 4y 2 )/2,

43

and this gives us

x=

(1 +

1 4y 2 )/2

and

x=

(1

1 4y 2 )/2

as the two nonnegative solutions. Hence:

1/2

((outer radius of washer)2 (inner radius of washer)2 ) dy

1 + 1 4y 2 2
1 1 4y 2 2
1/2

= 2
((
) (
) ) dy
2
2
0

W = 2
0

Now we consider the cylindrical shells method for both volumes.


about the

y -axis

When revolved

the red vertical rectangles generate the cylindrical shells that are

used in the computation of

by
x2 x4 . Hence:

W = 2 2 (radius
0

W,

and therefore the heights of these shells are given

of shell)(height of shell) dx

When revolved about the

x-axis

1
= 2 2 x x2 x4 dx
0

the green horizontal rectangles generate the

V . Therefore the heights

(1 1 4y 2 )/2. Hence:

cylindrical shells that are used in the computation of

of these shells are given by

V = 2 2
0

(1 +

1 4y 2 )/2

1/2

(radius of shell)(height of shell) dy

1 + 1 4y 2 1 1 4y 2
1/2

= 2 2
y(

) dy
2
2
0

Finally we compute the volumes. To compute

we use the integral we obtained

with the disk method,

x3 x5 1 4
V = 2 (x x ) dx = 2 [ ] =
3
5 0 15
0
1

44

and to compute

we use the integral we obtained with the washer method

W = 2
0
= 2
= 2
=

1/2

1/2
0
/2
0
/2

1+

1 4y 2 1

1 4y 2
) dy
2

1 4y 2 dy

cos

1
cos d
2

cos2 d

1 + cos 2
d
2
0
sin 2 /2
= [ +
]
2
4 0
2
=
4
=

/2

where we made the change of variable

Remark:
49.

Compare this example with

y=

1
1
sin , dy = cos d.
2
2

Example 36 in Part 2.

A water tank has a bottom consisting of a disk of radius

a with 0 a < 3, and a side surface


x = g(y)

having the shape generated by revolving the graph of a continuous nonnegative function
for

0y5

with

g(0) = a

and

g(5) = 3

about the

y -axis

where all units are in meters. Assume

that:

O1

As water runs out of a small hole at the bottom of tank, the speed of the water owing

through the hole at any moment is proportional to the square root of the depth of the water
in the tank at that moment.

O2

The function

and the constant

are chosen in such a way that the depth of the water

changes at a constant rate at all times.


Find the volume of the tank.

45

Solution:

Let

V (h)

denote the volume of the water when the depth is

V (h) =
Dierentiating this with respect to time

h.

Then:

g(y)2 dy

and using the Fundamental Theorem of

Calculus, Part 1 we obtain:

dV
dh
= g(h)2
dt
dt

O1
O2

dV /dt = k1 h for some positive constant


k1 and the condition
which says that dh/dt = k2 for some positive constant
k2 , we conclude that g(h) = k h1/4 for some positive constant k . In particular,
a = g(0) = 0, and g(5) = 3 gives k = 3/51/4 . Therefore g(h) = 3h1/4 /51/4 .

Using the condition

which says that

Hence the volume of the tank is:

V (5) =

5
0

3
51/4

1/4

9 y 3/2
) dy = 1/2
] = 30
3/2 0
5

46

3
m

50.

Let

be the volume of the water-dropper shown in the gure on the next page which has
x4 + y 4 = 1 about the line x = 5/2 where all units

the shape obtained by revolving the curve


are in centimeters.

a.

Express

as an integral using the cylindrical shells method.

b.

Express

as an integral using the washer method.

c.

Show that the improper integral

d.

Express

A=

in terms of

Solution: a.

1
0

3/4
1/4
0 u (1 u) du

u3/4 (1 u)1/4 du .
x (5/2)

The radius and height of the cylindrical shells are

(1 x4 )1/4 ((1 x4 )1/4 ),

and

respectively. Hence:

V = 2 (radius) (height) dx = 2

b.

converges.

5
(x + ) ((1 x4 )1/4 ((1 x4 )1/4 ) dx
2

The outer and inner radii of the washers are

5/2 + (1 y 4 )1/4

and

5/2 (1 y 4 )1/4 ,

respectively. Hence:

V =
=

c.

We have

u3/4
1

for all

1
1
1

((outer

2
radius)

(inner

2
radius) ) dy

5
5
(( + (1 y 4 )1/4 ) ( (1 y 4 )1/4 ) ) dy
2
2

0 < u 1 0 1 u < 1 0 (1 u)1/4 < 1 0 u3/4 (1 u)1/4 <


0 < u 1.

3/4
1/4
0 u (1 u) du

d.

Since

3/4
du
0 u

is convergent (because

by the Comparison Test.

We use the integral in part

(b) and symmetry


1

V = 10 (1 y 4 )1/4 dy
1
1

= 20 (1 y 4 )1/4 dy
0

= 20 (1 u)1/4
= 5

0
1

1 3/4
u
du
4

u3/4 (1 u)1/4 du

= 5A
as well as the change of variable

y = u1/4 , dy = 1/4 u3/4 du.

47

p = 3/4 < 1),

so is

48

51.

Find the absolute maximum and the absolute minimum values of

interval

[2, 2].

Solution:

f (x) = 2xex + (x2 3)ex = (x2 + 2x 3)ex .

f (x) = (x2 3)ex

We want to solve

on the

f (x) = 0.

(x2 + 2x 3)ex = 0 x2 + 2x 3 = 0 x = 1, 3 .
Since

[2, 2], the only


points x = 1,2, 2.

does not belong to the interval

Hence we are going to compute

at the

critical point is

f (1) = 2e, f (2) = e2 , and f (2) = e2 . Since e > 1 we have e2 > e2 > 2e.
the absolute maximum is e2 and the absolute minimum is 2e.

52.

Find the absolute maximum and the absolute minimum values of

Solution:

Let

y = x1/x .

Then

ln y =

obtain

ln x
.
x

x = 1.

Therefore

x1/x .

Dierentiating this with respect to

we

1
1 dy d
d ln x x x ln x 1 1 ln x
d 1/x
1 ln x
=
ln y =
=
=

x = x1/x
.
2
2
y dx dx
dx x
x
x
dx
x2
Since 1 ln x > 0 for 0 < x < e and 1 ln x < 0 for x > e, the absolute maximum value
of x1/x occurs at x = e and is e1/e . x1/x has no absolute minimum value.

Remark:

Although the reasoning above does not require it, let us also look at what happens

at the endpoints of the domain. Since


L'H

1/x
ln x
= lim
=0,
x 1
x x

lim ln y = lim

x
we have

53.

lim x1/x = lim y = lim eln y = e0 = 1.

Evaluate the limit

Solution:

xk =

lim (

2k/n
k=1

partition

1
n

On the other hand,

lim x1/x = 0

x0+

as

0 = 0.

1 n k/n
2 ).
n k=1
n

is a Riemann sum

k
, 0 k n,
n

f (ck ) xk

for

f (x) = 2x

on

[0, 1]

for the

k=1
and the sample points

ck =

k
, 1 k n.
n
1

1
1 n
2x
1
lim ( 2k/n ) = 2x dx =
] =
.
n n
ln 2 0 ln 2
0
k=1

49

Therefore,

Remark:

Alternatively, we can use the formula for the sum of a nite geometric series

2k/n =
k=1

2(n+1)/n 21/n
21/n
=
,
21/n 1
21/n 1

and then

1 n k/n
21/n
t2t
2t + t ln 2 2t
1
lim
=
lim
=
lim
.
=
2 ) = n
t
t
1/n
+
+
t0
n k=1
ln 2 2
ln 2
n(2 1) t0 2 1
L'H

lim (

54.

Evaluate the limit

Solution:
form

Let

1 .

y=(

form

Since

0
0

lim (
x0

sin x 1/x
) .
x

sin x
=1
x0 x

lim

1/x2

sin x
)
x

. Then

ln y =

and

ln (

1
= ,
x0 x2

lim

sin x
)
x
.
x2

As

this is limit has the indeterminate

x 0, this will have the indeterminate

and we can use L'Hpital's Rule.

ln(sin x) ln x
x0
x2
cos x 1

x cos x sin x

= lim sin x x = lim


x0
x0
2x
2x2 sin x
cos x x sin x cos x
sin x

= lim
= lim
2
x0 4x sin x + 2x cos x
x0 4 sin x + 2x cos x
sin x

1
1
x
= lim
=
=
x0 sin x
4+2
6
4
+ 2 cos x
x

lim ln y = lim
x0

L'H

L'H

Here applications of L'Hpital's Rule are indicated with


2

L'H

. Then

sin x 1/x
lim (
)
= lim y = lim eln y = e1/6
x0
x0
x0
x
using the continuity of the exponential function.

50

Remark:

The rst example of an application of L'Hpital's Rule in Guillaume Franois

Antoine Marquis de L'Hpital's book Analyse des Inniment Petits pour l'Intelligence des

Lignes Courbes of 1696:

There is a typo. Can you nd it?

55.

Evaluate

cos(2x) e2x
x0
sin4 x

lim

Solution:
cos(2x) e2x
cos(2x) e2x
x4
lim
=
lim
(

)
x0
x0
x4
sin4 x
sin4 x
2
cos(2x) e2x
x 4
= lim

(lim
)
x0
x0 sin x
x4
2
cos(2x) e2x
= lim
x0
x4
2
2 sin(2x) + 4xe2x

= lim
x0
4x3
2
2
4 cos(2x) + 4e2x 16x2 e2x

= lim
x0
12x2
2
2
8 sin(2x) 48xe2x + 64x3 e2x

= lim
x0
24x
sin(2x)
8
2
2
= lim (
2e2x + x2 e2x )
x0
3x
3
2
4
= 2+0=
3
3
2

L'H

L'H

L'H

Remark:

It is easier to solve this problem using the Taylor series, which will be seen in Calculus

51

II:

cos(2x) e2x
lim
x0
sin4 x

(2x)2 (2x)4 (2x)6


(2x2 )2 (2x2 )3
+

+ ) (1 + (2x2 ) +
+
+ )
2!
4!
6!
2!
3!
= lim
4
x0
x3
(x
+ )
3!
56
4
x4 + x6 +
45
= lim 3
2
x0
x4 x6 +
3
4 56 2
+ x +
= lim 3 45
2
x0
1 x2 +
3
4
=
3
(1

56.

Find the value of the constant

the limit for this value of

Solution:

sin(x + ax3 ) x
x0
x5

a for which the limit lim

a.

exists and compute

We have

sin(x + ax3 ) x
x0
x5
cos(x + ax3 )(1 + 3ax2 ) 1

= lim
x0
5x4
sin(x + ax3 )(1 + 3ax2 )2 + cos(x + ax3 )(6ax)

= lim
x0
20x3
cos(x + ax3 )(1 + 3ax2 )3 sin(x + ax3 ) 3(1 + 3ax2 )(6ax) + cos(x + ax3 )(6a)

= lim
.
x0
60x2
The numerator of the fraction inside this last limit goes to 1+6a as x 0. Therefore
the limit does not exist unless a = 1/6. If a = 1/6 then
lim
L'H

L'H

L'H

sin(x + x3 /6) x
x0
x5
cos(x + x3 /6)(1 + x2 /2)3 sin(x + x3 /6) 3(1 + x2 /2)x + cos(x + x3 /6)
= lim
x0
60x2

lim

is the sum of

cos(x + x3 /6)(1 (1 + x2 /2)3 )


x0
60x2
cos(x + x3 /6)(3/2 + 3x2 /4 + x4 /8)
1
= lim
=
x0
60
40

lim

and

sin(x + x3 /6) 3(1 + x2 /2)x


x0
60x2
sin(x + x3 /6)
1
(1 + x2 /2)(1 + x2 /6)
= lim

lim
=

.
x0
x0
x + x3 /6
20
20

lim

52

Hence

sin(x + x3 /6) x
3
= .
5
x0
x
40

lim

Remark:

Once again there are shorter ways of doing this. If we use the Taylor series then

(x + ax3 )3 (x + ax3 )5
+

3!
5!
1
1
1
= x + (a ) x3 + (
a) x5 +
6
120 2

sin(x + ax3 ) = (x + ax3 )

and it is immediate that the limit exists exactly when

a = 1/6

and then its value is

3/40.

57.

Let b > a > 0 be constants. Find the area of the surface generated by revolving the circle
(x b)2 + y 2 = a2 about the y -axis.

Solution:

We have

Surface Area

= 2

d
c

1 + ( dx ) dy
x
dy

for a surface generated

by revolving a curve about the y -axis. We have x = b +


2
2
a y and x = b a2 y 2 for the right and left halves of the circle, respectively.
Then

dx
dx
y
a

1+( ) =
.
=
2
2
2
dy
dy
a y
a y2
Hence

a
dy
a2 y 2 )
a
a2 y 2

a
a
+ 2 (b a2 y 2 )
dy
a
a2 y 2
a
1
= 4ab
dy
a
a2 y 2
a

Surface Area

= 2 (b +

y
= 4ab arcsin ( ) ]
a a

= 4ab (arcsin 1 arcsin(1)) = 4ab ( ( )) = 4 2 ab


2
2

53

Remark:

The surface generated by revolving a circle about a line (in the same plane) that

does not intersect it is called a torus.

58.

if

dy
= xy 2
dx

Solution:

We have

Find

y(1)

and

y(0) = 1.

dy
dy
dy
1 1
= xy 2 2 = x dx 2 = x dx = x2 + C .
dx
y
y
y 2
Then

Remark:
59.

y(0) = 1 1 = C .

Hence

y=

2
2 x2

. This in turn gives

What would your answer be if the question asked

y(1) = 2.

y(2)?

Nitrogen dioxide is a reddish-brown gas that contributes to air pollution, and also gives the

smog its color. Under sunlight it decomposes producing other pollutants, one of which is ozone.
As nitrogen dioxide decomposes, its density decreases at a rate proportional to the square of
the density. Suppose that the density of nitrogen dioxide
minutes and

6/25

grams per liter at time

t=3

1/2 grams per liter at time t = 0


Q when t = 15 minutes. (Assume

is

minutes. Find

that no new nitrogen dioxide is added to the environment.)

Solution:
k dt

and

dQ/dt = kQ2 where k is a positive


integrating we obtain 1/Q = kt + C where C
We have

constant. Then

dQ/Q2 =

is a constant.

t = 0 minutes we nd 1/(1/2) = 1/Q(0) = C , hence C = 2 liters per gram.


Now letting t = 3 min gives 1/(6/25) = 1/Q(3) = k 3 2 and hence k = 13/18
liters per gram per minute. Finally we obtain Q = 18/(13t + 36).
Letting

Using this we compute the density of nitrogen dioxide as

Q(15) = 6/77

grams per

liter after 15 minutes.

60.

f is a function that has a continuous second derivative and


f (0) = 4 , f (1) = 3 , f (0) = 5 , f (1) = 7, f (0) = 8 and f (1) = 11 . Show that:
Suppose that

that satises

0 f (x)f (x) dx 1

Solution:

du =

We rst do an integration by parts with


f (x) dx, v = f (x), to obtain:

u = f (x), dv = f (x) dx,

hence

f (x)f (x) dx = [f (x)f (x)] (f (x))2 dx


0
0

The rst term on the right is equal to f (1)f (1) f (0)f (0) = 3 7
the second term is nonnegative as (f (x))2 0. The result follows.

54

4 5 = 1,

and

Remark:

It can be shown that:

0 f (x)f (x) dx < 0

61.

Evaluate the following integrals.

a.
b.

3
sin x sin 2x dx
e

ln x
1 dx
x

Solution: a.

We use the identity

sin 2x = 2 sin cos x

to obtain

3
3
sin x sin 2x dx = sin x 2 sin x cos x dx

= 2 sin4 x cos x dx
= 2 u4 du

after the substitution

2
= u5 + C
5
2
= sin5 x + C
5
u = sin x, du = cos x dx.

b.

u=

The substitution

x, du = dx/(2 x)
e

ln x
dx = 4
1
x

gives:

ln u du

= 4 [u ln u u]

1
= 4 ( e ln e e + 1)

=42 e

Remark:

There are other ways of doing these. Here are some:

Solution: a.
substitution

Use the double-angle formula

u = 1 cos 2x, du = 2 sin 2x dx :

sin2 x = (1 cos 2x)/2


3

3
sin x sin 2x dx = (

1 cos 2x
) sin 2x dx
2

1
= u3/2 du
4 2
1
= u5/2 + C
10 2
1
= (1 cos 2x)5/2 + C
10 2
55

and then the

Or you can run this by simply saying

u = sin2 x, du = 2 sin x cos x dx = sin 2x dx

and:

3
3/2
sin x sin 2x dx = u du
2
= u5/2 + C
5
2
= sin5 x + C
5
Or use the identity

sin3 x = (3 sin x sin 3x)/4 and the trigonometric product to sum

formulas:

1
3
sin x sin 2x dx = 4 (3 sin x sin 3x) sin 2x dx
1
= (3 sin x sin 2x sin 3x sin 2x) dx
4
1
= (3(cos x cos 3x) (cos x cos 5x)) dx
8
1
= (cos 5x 3 cos 3x + 2 cos x) dx
8
1
1
1
=
sin 5x sin 3x + sin x + C
40
8
4
Or, if you are willing to go complex, use the identity

sin = (ei ei )/(2i) :

eix eix e2ix e2ix


sin
x
sin
2x
dx
=
(
)
dx

2i
2i
1
=
(e5ix + e5ix 3e3ix 3e3ix + 2eix + 2eix ) dx
16
eix eix
1 e5ix e5ix e3ix e3ix

+2
)+C
= (
16
5i
i
i
1
1
1
=
sin 5x sin 3x + sin x + C
40
8
4
3

b.

Do integration by parts:

ln x
dx = ln x d(2 x)
1
x
e dx

e
= [2 x ln x]1 2 x
x
1
e dx

= 2 e 2
1
x

e
= 2 e 4 [ x]1

=2 e4 e+4

=42 e

56

Or do the other integration by parts rst,

e 1
ln x
dx = d(x ln x x)
1
x
x
e
1 e x ln x x
x ln x x
=[
] +
dx
2 1
x3/2
x
1
e
1 e ln x
= 1 + dx [ x]1
2 1
x

1 e ln x
= 2 e + dx
2 1
x

and then from

1 e ln x
dx = 2 e

2 1
x

solve for

ln x
1 dx = 4 2 e .
x
e

Or rst do the substitution

x = eu , dx = eu du,

and then do an integration by parts:

e ln x
1 u
u
1 dx = 0 u e du
x
e

=
=

u eu/2 du

0
1
0

u d(2eu/2 )
1

= [2ueu/2 ]0 2
=

eu/2 du

0
1/2
u/2 1
2e 4[e ]0
1/2
1/2

= 2e

4e

=42 e

62.

+4

Evaluate the following integrals:

a.

dx

b.

c.

dx
(x2 + 1)2

1 x2 dx

Solution: a.

We rst do a change of variable

dx = 2 tet dt
57

x = t2 , dx = 2tdt,

and then do an integration by parts,

u = t, dv = et dt du = dt, v = et :

= 2tet 2 et dt
= 2tet 2et + C


= 2 xe x 2e x + C

b.
and

. Then dx = cos d
2
2

2
2
2
1 x = 1 sin = cos = cos = cos as cos 0 for .
2
2

1 x2 dx = cos cos d

We use the trigonometric substitution

x = sin ,

= cos2 d
1
(1 + cos 2) d
2
1
1
= + sin 2 + C
2
4
1
1
= + sin cos + C
2
2
1
1
= arcsin x + x 1 x2 + C
2
2
=

1
x

1 x2

c.
and

We use the trigonometric substitution

x = tan ,

x2 + 1 = tan2 + 1 = sec2 .

<< .
2
2

Then

sec2
dx
(x2 + 1)2 = (sec2 )2 d
= cos2 d
=
=
=
=
=

1
(1 + cos 2) d
2
1
1
+ sin 2 + C
2
4
1
1
+ sin cos + C
2
2
1
1
1
x

+C
arctan x +
2
2
x2 + 1
x2 + 1
1
1
x
arctan x + 2
+C
2
2 x +1
58

dx = sec2 d

x2 + 1

Remark:

Other methods can also be used. For instance, the integral in part

(b) can be done

using integration by parts.

63.

Evaluate the following integrals:

1/2

a.

1/2

b.

1/2

c.

1/2

1x
arcsin x dx
1+x

dx

x + 1 x2

dx
1 + ex

Solution: a.
1/2

1/2

We rst re-write the integral as follows:

1/2
1/2
1x
1
x

arcsin x dx =
arcsin x dx +
arcsin x dx
2
1+x
1/2
1/2
1x
1 x2

The rst integral on the right vanishes as the integrand is odd and the integration
interval is symmetric about the origin. We do integration by parts for the second
integral with

u = arcsin x

and

dv =

1 x2 :
1/2

1/2

arcsin x

x
1 x2

1 x2

dx = [arcsin x

Therefore:

1/2

1/2

dx,

and hence

1 x2 ]

1/2
1/2

1/2
1/2

1x

arcsin x dx = 1
1+x
2 3

59

du =

1
1 x2

and

dx = 1
2 3

v=

b.

We start by changing variables

1/2

1/2

c.

Let

t = sin , dt = cos d.

/6
dx
cos d

=
2
/6 sin + cos
x+ 1x
/6 cos (cos sin )
=
d
/6
cos2 sin2
1 /6 1 + cos 2 sin 2
=
d
2 /6
cos 2
1 /6
=
(sec 2 + 1 tan 2) d
2 /6
/6
1
1 1
= [ ln tan 2 + sec 2 + ln sec 2]
2 2
2
/6

1
= ln( 3 + 2) +
2
6

u = ex + 1, du = ex dx.

Then

dx
ex dx
du
x
=
1 + ex ex + 1 = u = ln u + C = ln(1 + e ) + C
and from this

c dx
dx
=
lim
= lim [ln(1 + ex )]c0 = lim (ln(1 + ec ) ln 2) = ln 2
c
c
1 + ex c 0 1 + ex

follows.

Remark:

Here is another way of doing part

(c).

Let

u = ex , du = ex dx.

dx
ex dx
1 + ex = ex + (ex )2
du
=
u + u2
1
1
= (
) du
u 1+u
= ln u ln 1 + u + C
= x ln(1 + ex ) + C
and from this

c dx
dx
=
lim

1 + ex c 0 1 + ex
= lim [x ln(1 + ex )]c0
c

= lim (c ln(1 + ec ) + ln 2)
c

= lim ( ln(ec ) ln(1 + ec ) + ln 2)


c

= lim ( ln(ec + 1) + ln 2)
c

= ln 2
60

Then

follows.

64.

Evaluate the improper integral

0
where

dx
(ax + 1)(x2 + 1)

is a positive constant.

Solution:

We have the partial fraction decomposition

1
1
a2
ax 1
=
(
2
).
2
2
(ax + 1)(x + 1) a + 1 ax + 1 x + 1
Hence:

dx
1
a2
ax 1
=
(
(ax + 1)(x2 + 1) a2 + 1 ax + 1 dx x2 + 1 dx)
=

a2

1
a
(a ln ax + 1 ln(x2 + 1) + arctan(x)) + C .
+1
2

Therefore:

c
dx
dx
=
lim

2
(ax + 1)(x + 1) c 0 (ax + 1)(x2 + 1)
c
1
a
2
= 2
lim [a ln ax + 1 ln(x + 1) + arctan(x)]
a + 1 c
2
0

1
a
lim
(a
ln(ac
+
1)

ln(c2 + 1) + arctan c
2
c
a +1
2
a ln 1 +

=
=
as

65.

ac + 1
lim
=a
c
c2 + 1

The curve

and

a2

a
1
ac + 1
)+ 2
lim ln (
lim arctan c
c
2
+1
a + 1 c
c +1

a2

(a ln a + )
+1
2

is revolved about the

region between the curve and the


solid

.
2

lim arctan c =

y = 1/x, x 1,

x-axis

for

x1

x-axis

to generate a surface

is revolved about the

D.

a.

Show that

has nite volume.

b.

Show that

has innite area.

Solution: a.

a
ln 1 arctan 0)
2

Using the disk method we obtain


Volume

R(x)2 dx =
61

dx
.
x2

x-axis

and the

to generate a

Since

b.

p = 2 > 1,

this integral converges.

The surface area formula gives

Surface Area

We have

1
x

1+

= 2

1 + (y )2 dx = 2

dx
1 1

0
for x 1, and
= .
x4 x
x
1

1
x

1+

1
dx .
x4

The surface area is innite

by the Direct Comparison Test.

Remark:

We want to get the surface

painted for a reasonable, nite price. We oer the job

to Painter1 and Painter2.

Painter1 says: It cannot be done.

has innite area, it cannot be painted with

nite amount of paint.

Painter2 says: It can be done.

has nite volume. We can ll the inside of

with volume(D ) cubic units of paint and let the excess paint run out.
Whom should we believe?

66.

Let

be a nonnegative integer. Show that

Solution:

We use induction on

Let

n = 0.

Let

n>0

Then

tn et dt = n! .

n.

et dt = 1 = 0! .

and assume that

tn1 et dt = (n 1)! .

Integration by parts gives

tn et dt = lim tn et dt
c 0
= lim ([tn et ]0 + n

= lim [tn et ]0 + n
c

= n

tn1 et dt)
tn1 et dt

n1 t

e dt

= n (n 1)!
= n!
where

Remark:

lim cn ec = 0

can be seen after

applications of L'Hpital's Rule.

The Gamma function is dened by

(x) =

62

tx1 et dt

for

x > 0.

x > 0.

It can be shown that the improper integral on the right converges if and only if

Note that 0 is also a bad point besides

for

0 < x < 1.

A calculation similar to the one in the solution above shows that


This relation

(x) = (x + 1)/x

(x + 1) = x(x)

for

x > 0.

can be used repeatedly to dene the Gamma function for all

real numbers which are not nonpositive integers.


Since

(n + 1) = n! for all nonnegative integers n, we can use the Gamma function to dene the
x! = (x + 1). In particular,

1

1/2 t
u2
( )! = (1/2) = t e dt = 2 e du = 2
= .
2
2
0
0

factorials of all real numbers which are not negative integers by

The volume of an

n=1

(the interval

The case

67.

n=4

Show that

n-dimensional ball with radius r is n/2 rn /(n/2)! . Check this formula for
[r, r]), n = 2 (the disk with radius r), and n = 3 (the sphere with radius r).

will be seen in Calculus II.

Solution:

ln x
dx = 0 .
x2 + 1

First of all, by denition,

1 ln x
ln x
ln x
dx
=
dx
+
0 x2 + 1
1 x2 + 1 dx
x2 + 1

and the improper integral on the left converges if and only if both basic improper
integrals on the right converge.

Consider

ln x
dx .
x2 + 1
1

Note that

ln x
ln x
2
2
x +1
x

for

x 1.

On the other hand,

ln x
t
dx
=
0 te dt
x2

= lim
c

= lim
c

tet dt

0
c
0

t d(et )
c

= lim ([ tet ]0 +
c

=
=

et dt)

0
t c
lim (ce [e ]0 )
c
lim (cec ec + 1)
c
c

=1
where we used the substitution

x = et , dx = et dt,

followed by the denition of the

improper integral, then an integration by parts and nally the limit


L'H

1
c
= lim c = 0 .
c
c e
c e

lim cec = lim

Therefore by the Direct Comparison Test the improper integral


converges.

63

ln x
dx
x2 + 1

Now the convergence of

1 ln x
0 x2 + 1 dx

follows as

1 ln x
1 ln(1/u)
ln u
du
0 x2 + 1 dx = (1/u)2 + 1 u2 = 1 1 + u2 du
where we used the change of variable

x = 1/u, dx = du/u2 ,

and this also gives:

1 ln x
ln x
ln x
dx
=
dx
+
0 x2 + 1
1 x2 + 1 dx
x2 + 1
ln x
ln x
=
dx
+
1 x2 + 1 dx = 0
x2 + 1
1

Remark:

Instead of computing

comparison

ln x
1
3/2
2
x
x

for

x1

ln x
dx,
x2

one can show its convergence using the

and the fact that

dx
x3/2

is convergent as

p = 3/2 > 1.

1
ln x

for x 1 can be seen as follows: Consider f (x) =


x ln x on
x2
x3/2

[1, ). Then f (x) = 1/(2 x) 1/x = ( x 2)/(2x)


and x = 4 is the only critical point. Since

f (x) < 0 for x < 4 and f (x) > 0 for x > 4, f (4) = 4 ln 4 = 2 2 ln 2 = 2(1 ln 2) > 0 must be
the absolute minimum value of f on [1, ), and we are done.
Here the fact that

68.*

Determine whether the improper integral

Solution:

dx
ex ex

converges or diverges.

By denition,

dx
1
dx
=
+

x
x
x
x
x
e e
e ex
0 e e
1

and the given integral converges if and only if both of the integrals on the right
hand side converge.

Let us consider

1
dx
0 ex ex

rst. Since we have the linearization

ex ex (1 + x) (1 x) = 2x ,
centered at

x = 0,

we expect

1/(ex ex )

to behave like

1/(2x)

near the bad point

0, and therefore this integral to diverge.


In fact, we have

1
1/(ex ex )
1
x

= lim+ x x = lim+ x x = .
x0 e + e
x0 e e
1/x
2
L'H

L = lim+
x0

1 dx
0 < L < and
is divergent (because p = 1 1), we conclude
x
0

1
dx
dx
diverges
by
the
Limit
Comparison
Test.
Therefore
0 ex ex
0 ex ex

Since
that

diverges too.

*Examples marked red are not part of the Fall 2015 syllabus.
64

Remark:

The other improper integral on the right hand side converges. We have

1/(ex ex )
1
=
lim
= 1.
x
x 1 e2x
ex

L = lim
Since

0<L<
1

and

we conclude that

69.*

ex dx = lim ex dx = lim [ex ]c1 = lim (ec + e1 ) = e1 < ,


c 1
c
c
1

ex

dx
ex

converges by the Limit Comparison Test.

Determine whether the improper integral

Solution:

1 e1/x

dx
x

converges or diverges.

By denition,

1 1 e1/x
1 e1/x
1 e1/x

dx =
dx +
dx
0
1
x
x
x

and the given integral converges if and only if both integrals on the right hand side
converge.
On one hand, since

1 e1/x

x
L = lim+
= lim+ (1 e1/x ) = 1
1
x0
x0

x
1

is a positive real number, and

1 e1/x

dx
x

dx
0
x

converges as

p = 1/2 < 1;

we conclude that

converges by the Limit Comparison Test.

On the other hand, since

1 e1/x

1 e1/x
1 et
et
x
= lim
L = lim
= lim+
= lim+
=1
x
x
1
t0
t0 1
1/x
t
x3/2
L'H

is a positive real number, and

1 e1/x

dx
x

Hence

dx
x3/2

converges as

p = 3/2 > 1;

also converges by the Limit Comparison Test.

1 e1/x

dx
x

converges.

*Examples marked red are not part of the Fall 2015 syllabus.
65

we conclude that

Part 2: Multi-variable Functions

1.

Consider the point

P (3, 5, 1)

L x = 2t 1 , y = t + 2 , z = 2t ; < t < .

and the line

a.

Find the equation of the plane passing through

perpendicular to

b.

Find the equation of the plane passing through

and containing

Solution: a.

L.

L.

v = 2i j 2k . Therefore we can
take the normal vector of the plane to be n = 2i j 2k . Then the equation of the
plane is 2 (x 3) + (1) (y (5)) + (2) (z 1) = 0 , or 2x y 2z = 9 .

b.

The line

is parallel to the vector

n to the plane containing the line L and the point P will be


#
v, and it will also be perpendicular to P Q where Q is any point
on the line. We can take Q(1, 2, 0), the point corresponding to t = 0 , and then
#
P Q = 4i + 7j k . Now we can take the normal vector v to be
A normal

perpendicular to

RR i
j kRRRR
# RRRR
R
n = v P Q = RRR 2 1 2 RRRR = 15i + 10j + 10k ,
RRR
RR
RR4 7 1 RRR
or in fact

n = 3i + 2j + 2k .

Then the equation of the plane is

3 (x 3) + 2 (y (5)) + 2 (z 1) = 0 ,
or

2.

3x + 2y + 2z = 1 .

Consider the plane

P 3x 4y + z = 10 ,

and the points

a.

Find the equation of the line passing through

b.

Find the equation of the plane passing through

Solution: a.

Since

n = 3i 4j + k

P (2, 3, 1)

perpendicular to

and

b.

P.
P.

is normal to the plane, it will be parallel to any

v = 3i 4j + k ,
x = 3t + 2, y = 4t + 3, z = t 1; < t < .

A plane perpendicular to

Q(1, 2, 2) .

perpendicular to

line perpendicular to the plane. Hence we can take


of the line is

and

will have a normal

and the equation

perpendicular to the normal

n = 3i 4j + k of P . Also a plane containing the points P and Q will have a normal


#
n perpendicular to P Q = i j + 3k . Therefore we can take n to be
RR i
j
# RRRR
R
n = n P Q = RR 3 4
RR
RRR1 1

66

kRRRR
R
1 RRRR = 11i 10j 7k ,
R
3 RRRR

or rather

n = 11i + 10j + 7k .

This gives the equation of the plane as

11 (x 2) + 10 (y 3) + 7 (z (1)) = 0 ,
or

3.

11x + 10y + 7z = 45 .

Find a parametric equation of the line

that intersects both of the lines

L1 x = 2t 1,

y = t + 2,

z = 3t + 1

L2 x = s + 5,

y = 2s + 3,

z = s

and

perpendicularly.

Solution: v1 = 2i j + 3k and v2 = i + 2j k are the velocity vectors of the lines L1


and

L2 ,

respectively. Hence,

is a velocity vector for

RRR i
j kRRRR
RR
R
v = RRRR2 1 3 RRRR = 5i + 5j + 5k
RRR
R
RR1 2 1 RRRR
L. So we may take v = i j k.

Then

RR i
j kRRRR
RR
RRR
R
n = vv1 = RR1 1 1 RRRR = 4i 5j + k
RRR
RRR
RR2 1 3 RR
is normal to the plane P containing the lines L and L1 . As P1 (1, 2, 1)
an equation of P is

is in

P,

4 (x (1)) + (5) (y 2) + 1 (z 1) = 0 ,
or

4x + 5y z = 5.

At the point

P0

of intersection of

and

L2 , s

satises:

4 (s + 5) + 5 (2s + 3) (s) = 5
Hence

s = 2.

Substituting this back in the equations of

Therefore an equation of

x = t + 3,

L2

gives

P0 (3, 1, 2).

is:

y = t 1 ,

z = t + 2 ;

67

( < t < ) .

4.

c be a constant. Show the angle between the position and the velocity vectors along the
r = ect cos t i + ect sin t j , < t < , is constant.

Let

curve

Solution:

We have

r = ect cos t i + ect sin t j


and

v=

dr
= (c ect cos t ect sin t)i + (c ect sin t + ect cos t)j .
dt

Then

r = ((ect cos t)2 + (ect sin t)2 )1/2 = ect ,

v = ((c ect cos t ect sin t)2 + (c ect sin t + ect cos t)2 )1/2 = c2 + 1 ect ,
r v = ect cos t (c ect cos t ect sin t) + ect sin t (c ect sin t + ect cos t) = c e2t .
Therefore, if

is the angle between

cos =
and we conclude that

and

v,

we have

rv
c e2t
c

=
=
,
ct
2
ct
2
r v e c + 1 e
c +1

is constant.

68

5.

In the

xyz -space

where a yscreen lies along the plane with the equation

2x + y 2z = 1 ,
the trajectory of a bee as a function of time

is given by

r = t i + t2 j + t3 k
for

< t < .

a.

Find all times

when the bee is ying parallel to the screen.

b.

Find all times

when the bee is ying perpendicular to the screen.

c.

There are holes in the screen through which the bee passes. Find the coordinates of all

of these holes.

Solution: a.

2i + j

The normal vector of the plane along which the screen lies is n =
2k, and the velocity vector of the bee is v = i + 2tj + 3t2 k. The bee is ying

parallel to the screen whenever these two vectors are perpendicular to each other;
in other words, whenever n v = 0. As n v = 2 + 2t 6t2 , we want 3t2 t 1 = 0.
Hence

b.

t = (1

13)/6

are the times when the bee is ying parallel to the screen.

The bee is ying perpendicular to the screen whenever n and v are parallel
1/2 = 2t/1 = 3t2 /(2). This implies t2 =

to each other; in other words, whenever

1/3

which is not possible. Therefore there is no moment when the bee is ying

perpendicular to the screen.

c.

Substituting the coordinates

x = t, y = t2 , z = t3

of the position of the bee


of the plane we obtain 2t3 t2 2t + 1 = 0. As

2x + y 2z = 1
(2t 1)(t 1)(t + 1), the y is in the plane of the screen when
t = 1/2, t = 1, t = 1. These times correspond to the points (x, y, z) = (1/2, 1/4, 1/8),
(1, 1, 1) and (1, 1, 1).
into the equation
2t3 t2 2t + 1 =

69

6. a.

Show that

b.

Show that

xy 2
=0.
(x,y)(0,0) x6 + y 2
xy
lim
does
6
(x,y)(0,0) x + y 2

Solution: a.

lim

We have

0 y 2 x6 + y 2

0
for all

(x, y) =/ (0, 0) .

not exist.

for all

(x, y).

Hence

xy 2
y2

=
x

x 1 = x
x6 + y 2
x6 + y 2

Since

lim

(x,y)(0,0)

x = 0 ,

the Sandwich Theorem gives

xy 2
=0.
(x,y)(0,0) x6 + y 2
lim

b.

The limit along the

x-axis

lim

(x,y)(0,0)
x-axis

along the

whereas the limit along the

lim

xy
x0
= lim 6
= lim 0 = 0 ,
2
x0 x + 02
x0
+y

x6

y=x

(x,y)(0,0)
y=x

along the line

is

line is

xy
xx
1
= lim 6
= lim 4
=1.
2
2
x0 x + x
x0 x + 1
+y

x6

Since these two limits are dierent, the two-variable limit

lim

xy
+ y2

(x,y)(0,0) x6

does not

exist by the Two-Path Test.

Remark:
Let

Let

and

Sertz Theorem tells exactly when such limits exist:

be nonnegative real numbers, and

and

If

a b
+ >1,
c d

then

xa yb
=0.
(x,y)(0,0) xc + yd

If

a b
+ 1,
c d

then

xa yb
(x,y)(0,0) xc + yd

and

be positive real numbers.

lim
lim

be nonnegative integers, and

does not exist.

and

If

a b
+ >1,
c d

then

xa y b
=0.
(x,y)(0,0) xc + y d

If

a b
+ 1,
c d

then

xa y b
(x,y)(0,0) xc + y d

be positive even integers.

lim
lim

does not exist.

70

http://www.fen.bilkent.edu.tr/otekman/calc2
/sertoztheorem.pdf. A proof of the n-variable case of the second version can be found in
http://sertoz.bilkent.edu.tr/depo/limit.pdf.
A proof of the rst version can be found in

7.

Determine all values of the constant

>0

for which the limit

x2 y 3
(x,y)(0,0) x3 + y
lim

exists.

Solution:

We observe that

lim

(x,y)(0,0)
x = y3

along the curve

1
y0 1 + y9

lim

is

if

1
(y 3 )2 y 3
y
x2 y 3
=
lim
= lim lim
.
3

3
3

y0 y + y
y0 y y0 1 + y9
x + y

>9

and

exist. Therefore the limit of

1/2

= 9.

if

x2 y 3
x3 + y

consequently the two-variable limit

along the curve

x2 y 3
(x,y)(0,0) x3 + y
lim

Now we will show that the limit is 0 if

On the other hand

< 9.

x = y3

y
y0 y

lim

does not

does not exist, and

does not exist either for

9.

We have

x2 y 3
(x/y/3 )2
y3/3 y3/3

=
x3 + y
(x/y/3 )3 + 1

Here we used the fact that if t 1, then t2 t3 and hence


t2

1
, and if 0 < t < 1, then t2 < 1 gives
1. Since
lim y3/3 = 0 for
(x,y)(0,0)
t3 + 1
t3 + 1
x2 y 3
< 9,
lim
= 0 follows by the Sandwich Theorem.
(x,y)(0,0) x3 + y

for all
t2

8.

(x, y) =/ (0, 0).

Determine all values of the positive constant

lim

(x,y)(0,0) (x2

for which the limit

x
+ y 2 )k

exists.

Solution:
Suppose

The limit exists for

k<

1
2

. We have

k<

1
2

, and does not exist for

0 x2 x2 + y 2

for all

(x, y),
k

and hence

x2
x
12k

x
(
) x12k
0 2
(x + y 2 )k
x2 + y 2
71

1
2

for all

(x, y) =/ (0, 0) .

1 2k > 0,

Since

Therefore

lim

(x,y)(0,0) (x2

we have

x12k 0

as

(x, y) (0, 0).

x
=0
+ y 2 )k

by the Sandwich Theorem.

k=

Suppose

1
2

. Then

lim

(x,y)(0,0)
x-axis

along the

(x2

x
x
= lim 2
= lim 1 = 1 ,
2
1/2
x0 (x + 02 )1/2
x0
+y )

(x2

0
x
= lim 2
= lim 0 = 0 .
2
1/2
y0 (0 + y 2 )1/2
x0
+y )

and

lim

(x,y)(0,0)
y -axis

along the

Since these limits are dierent, the two-variable limit

lim

(x,y)(0,0) (x2

x
+ y 2 )1/2

does not

exist by the Two-Path Test.

k > 1/2 .

Suppose

Then

lim

(x,y)(0,0)
x-axis

along the

(x2

x
x
= lim 2
= lim x12k =
2
k
x0 (x + 02 )k
x0
+y )

does not exist. Therefore the two-variable limit

x
(x,y)(0,0) (x2 + y 2 )k
lim

does not exist

either.

9.

Let

where
of

xa y b

x4 + y 6
f (x, y) =

0
a

and

a.

and

(x, y) =/ (0, 0)

if

(x, y) = (0, 0)

are nonnegative integers. In each of

for which

f (x, y)

if

satises the given condition.

is continuous at

(0, 0).

b.

as

f (x, y) goes to 1 as (x, y) approaches (0, 0)


(x, y) approaches (0, 0) along the line y = x.

c.
limit

(a-e), determine whether there exist values

f (x, y) goes to 0 as (x, y) approaches (0, 0)


lim f (x, y) does not exist.

along the line

y = x,

and

goes to

along any line through the origin, and the

(x,y)(0,0)

d.

f (x, y) goes to 0 as (x, y) approaches (0, 0) along any line through


y -axis, and f (x, y) goes to 1 as (x, y) approaches (0, 0) along the y -axis.

e.

f (x, y)

fx (0, 0)

and

fy (0, 0)

exist, and

f (x, y)

is not dierentiable at

72

the origin except the

(0, 0).

Solution: a.

a=4

If

and

b = 1,

then

is continuous at

(0, 0).

This follows from

the Sandwich Theorem as

0 f (x, y) =
for

(x, y) =/ 0

b.

Let

a=3

x4 y
x4

y 1 y y
x4 + y 6
x4 + y 6

implies that the limit of

b = 1.

and

f (x, y)

at

(0, 0)

0 = f (0, 0).

is

Then

lim

(x,y)(0,0)
y=x

x4
1
= lim
=1
4
6
x0 x + x
x0 1 + x2

f (x, y) = lim f (x, x) = lim


x0

along the line

and

lim

(x,y)(0,0)
y=x

x4
1
=
lim
= 1 .
x0 x4 + x6
x0 1 + x2

f (x, y) = lim f (x, x) = lim


x0

along the line

c.

Let

a=2

and

b = 3.

lim

(x,y)(0,0)
y=mx

Then we have

m3 x5
m3 x
=
lim
=0
x0 x4 + m6 x6
x0 1 + m6 x2

f (x, y) = lim f (x, mx) = lim


x0

along the line

as well as

lim

(x,y)(0,0)
along the y -axis

f (x, y) = lim f (0, y) = lim 0 = 0 .


y0

y0

However,

lim

(x,y)(0,0)
y=x2/3

x4
1 1
= lim = =/ 0
x0 x4 + x4
x0 2
2

f (x, y) = lim f (x, x2/3 ) = lim


x0

along the curve

and hence the limit of

d.

Let

a=0

and

f (x, y)

b = 6.

lim

(x,y)(0,0)
y=mx

at

(0, 0)

does not exist by the 2-Path Test.

Then we have

m6 x6
m6 x2
=
lim
=0
x0 x4 + m6 x6
x0 1 + m6 x2

f (x, y) = lim f (x, mx) = lim


x0

along the line

and

lim

(x,y)(0,0)
y -axis

y6
= lim 1 = 1 .
y0
y0 y 6

f (x, y) = lim f (0, y) = lim


y0

along the

e.

If

a=1

and

b = 1,

continuous there.

then

f (x, y)

is not dierentiable at

(0, 0)

as it is not even

This can be seen by considering its limit along the line

which does not exist. On the other hand,

fx (0, 0)

and

fy (0, 0)

y =x
f is

are both 0 as

identically zero on both axes.

Remark:

The complete lists of ordered pairs

(a, b)

given conditions are as follows:

73

of nonnegative integers that satisfy the

a.

all

b.

(3, 1), (1, 3)

c.

(1, 4), (2, 3)

d.

(0, 6)

e.

(0, 7), (1, 1), (1, 2), (1, 3), (1, 4), (1, 5), (2, 1), (2, 2), (2, 3), (2, 4), (3, 1), (3, 2), (4, 1)

10.

(a, b)

with

3a + 2b > 12

In Genetics, Fisher's Equation ,

p
2p
= p (1 p) + 2
t
x
describes the spread of an advantageous allele in a population with uniform density along a
1-dimensional habitat, like a shoreline, as a result of both reproduction and dispersion of the
ospring. Here

p(x, t) is the frequency of the allele as a function of the position x and the time

t.
Find all possible values of the pair of constants

p(x, t) =

(a, b)

for which the function

1
(1 + eax+bt )2

satises the Fisher's Equation.

Solution:

We have

pt = 2(1 + eax+bt )3 eax+bt b


px = 2(1 + eax+bt )3 eax+bt a
pxx = 6(1 + eax+bt )4 (eax+bt a)2 2(1 + eax+bt )3 eax+bt a2
p(1 p) = (1 + eax+bt )4 (2eax+bt + (eax+bt )2 )
and substitution these in the Fisher's Equation gives

2b(1 + eax+bt ) = 2 + eax+bt + 6a2 eax+bt 2a2 (1 + eax+bt )


or:

2a2 2b 2 = (4a2 + 2b + 1)eax+bt


As the left hand side if this equality is constant, so must be the right hand side. This
is possible only if eax+bt is constant or 4a2 + 2b + 1 = 0. eax+bt is constant only if a = 0

and

b = 0,

and for these values the equation becomes 2 = 1 which is not possible.
In the second case, the equation becomes 2a2 2b 2 = 0. Adding 4a2 + 2b + 1 = 0 and

2a2 2b 2 = 0 we nd 6a2 = 1 and hence b = 5/6. Therefore (a, b) = (1/ 6, 5/6)
and

(1/ 6, 5/6)

are the only values for which the given function satises the

Fisher's Equation.

74

11.

Assume that

yz 2 + z 3 = 1

and

Solution:
z

and

and

are dierentiable functions of

zw3 xz 3 + y 2 w = 1 .

Find

z
x

at

x and y

x,

but

xw3 +

(x, y, z, w) = (1, 1, 1, 1) .

Dierentiating the equations with respect to

depend on

satisfying the equations

x and keeping in mind that

does not; we obtain

w3 + x 3w2 wx + y 2zzx + 3z 2 zx = 0 ,
and

zx w3 + z 3w2 wx z 3 x 3z 2 zx + y 2 wx = 0 .
Substituting
Solving for

x = 1, y = 1, z = 1, w = 1,

zx

5zx + 3wx = 1

and

2zx + 2wx = 1 .

we nd

z
5
=
x
4

12.

we get

Once upon a time there was an

at

(x, y, z, w) = (1, 1, 1, 1) .

xy -plane. The temperature at each point of this xy -plane


xy -plane. Each bug had a device that measured the

changed as time passed. Bugs roamed this

temperature in real time, and showed the rate of change of temperature with respect to time
on its screen.
One day four of these bugs met at a point

P0 .

The rst bug was moving with velocity

At the moment they met:

v1 = 2i + j

The second bug was moving with velocity


C/s.
The third bug was moving with velocity

v2 = i 5j

v3 = i + j

The fourth bug was sitting still.

What was the device of the fourth bug showing?

75

m/s and its device was showing

1 C/s.

m/s and its device was showing

m/s and its device was showing

2 C/s.

Solution:

13.

Let

Suppose

z = f (x, y)

be a dierentiable function such that

f (3, 3) = 1,

fx (3, 3) = 2,

fy (3, 3) = 11,

f (2, 5) = 1,

fx (2, 5) = 7,

fy (2, 5) = 3.

is a dierentiable function of

and

satisfying the equation

f (w, w) = f (uv, u2 + v 2 )
for all

(u, v).

Find

Solution:

w
u

at

(u, v, w) = (1, 2, 3).

Dierentiating the identity

f (w, w) = f (uv, u2 + v 2 )
with respect to

fx (w, w)

gives

w
w
(uv)
(u2 + v 2 )
+ fy (w, w)
= fx (uv, u2 + v 2 )
+ fy (uv, u2 + v 2 )
u
u
u
u
76

by the Chain Rule. Hence

(fx (w, w) + fy (w, w))

w
= fx (uv, u2 + v 2 ) v + fy (uv, u2 + v 2 ) 2u
u

which leads to

w
= 2fx (2, 5) + 2fy (2, 5)
u
after substituting (u, v, w) = (1, 2, 3).
Now using fx (3, 3) = 2, fy (3, 3) = 11,
fx (2, 5) = 7, and fy (2, 5) = 3, we conclude that
(fx (3, 3) + fy (3, 3))

w 8
=
u 9

14.

Let

at

(u, v, w) = (1, 2, 3) .

u = x + y + z , v = xy + yz + zx, w = xyz , and suppose that f (u, v, w) is


f (u, v, w) = x4 + y 4 + z 4 for all (x, y, z). Find fu (2, 1, 2).

a dierentiable

function satisfying

Solution:

We can take

Dierentiating

(x, y, z) = (1, 1, 2)

as this gives

(u, v, w) = (2, 1, 2).

f (u, v, w) = x4 +y 4 +z 4 with respect to x, y , z , respectively, we obtain:


fu ux + fv vx + fw wx = 4x3
fu uy + fv vy + fw wy = 4y 3
fu uz + fv vz + fw wz = 4z 3

Now using

u = x + y + z , v = xy + yz + zx, w = xyz ,

these give:

fu 1 + fv (y + z) + fw yz = 4x3
fu 1 + fv (x + z) + fw xz = 4y 3
fu 1 + fv (x + y) + fw xy = 4z 3
Substituting

(x, y, z) = (1, 1, 2)

we get:

fu + fv 2fw = 4
fu + 3fv + 2fw = 4
fu fw = 32
Subtracting 3 times the rst equation from the second gives
adding 8 times the third equation to this gives

Remark:
for a given

the

So

2fu + 8fw = 16, and


fu (2, 1, 2) = 40.

f (u, v, w) = u4 4u2 + 2v 2 + 4uw is one. Also note that


corresponding (x, y, z) must be the roots of T 3 uT 2 + vT w = 0 and

There are such

(u, v, w),

6fu = 240.

f.

In fact,

hence is determined up to a permutation of its entries, making the answer independent of the
choice.

15.

Let

z = f (x, y)

be a twice-dierentiable function and

x = r cos , y = r sin .

2f 2f 2z 1 z 1 2z
+
=
+
.
+
x2 y 2 r2 r r r2 2

77

Show that

Solution:

If

z = F (x, y)

y,

then by the chain

z F x F y
=

= Fx cos + Fy sin ,
r x r y r

(i )

z F x F y
=

= Fx (r sin ) + Fy (r cos ) .
x y

(ii )

rule we have

is a dierentiable function of

and

and similarly,

We use

(i ) with F

=f

to obtain

z
= fx cos + fy sin .
r
Then

To compute

2z

= (fx ) cos + (fy ) sin .


2
r
r
r

(fx ) and
(fy ) we use (i ) with F = fx and F = fy ,
r
r

respectively:

2z
= (fxx cos + fxy sin ) cos + (fyx cos + fyy sin ) sin
r2
= fxx cos2 + 2fxy cos sin + fyy sin2
Similarly, using

(ii ) with F

=f

A)

B)

gives

z
= fx (r sin ) + fy (r cos ) ,

and dierentiating this with respect to

again gives

2z

= (fx ) (r sin ) + fx (r sin )


2

+ (fy ) (r cos ) + fy (r cos )

= (fxx (r sin ) + fxy r cos )(r sin ) + fx (r cos )


+ (fyx (r sin ) + fyy r cos )(r cos ) + fy (r sin )
2
2
= fxx r sin 2fxy r2 cos sin + fyy r2 cos2
r(fx cos + fy sin )

16.

where we used

(ii ) with F

Now if we add

(B), 1/r times (A), and 1/r2 times (C), we obtain fxx + fyy .

Suppose that

= fx

C)

F = fy .

f (x, y) is a twice-dierentiable function with continuous derivatives satisfying


f(

(x, y) =/ (0, 0).


5, fyy (3, 1) = 4.

for all

and

Find

x2

y
x
, 2
) = f (x, y)
2
+ y x + y2

fxx (3/10, 1/10)

if

fx (3, 1) = 8, fy (3, 1) = 7, fxx (3, 1) = 2, fxy (3, 1) =

78

Solution:

We have:

fx (x, y) =

x
y
f( 2
,
)
x x + y 2 x2 + y 2

= fx (

x2

x
x
, 2
)
( 2
)
2
2
+y x +y
x x + y 2
+ fy (

= fx (

x2

x
y
y

( 2
, 2
)
)
2
2
+y x +y
x x + y 2

x
y
1 (x2 + y 2 ) x 2x
,
)

x2 + y 2 x2 + y 2
(x2 + y 2 )2
+ fy (

x2

x
y
2xy
, 2
) 2
2
2
+y x +y
(x + y 2 )2

Dierentiating this a second time we obtain:

fxx (x, y) =

x
y
y 2 x2
(fx ( 2
,
)

x
x + y 2 x2 + y 2 (x2 + y 2 )2
+ fy (

x2

x
y
2xy
, 2
) 2
)
2
2
+y x +y
(x + y 2 )2

x
y
y 2 x2
= fxx ( 2
,
)

(
)
x + y 2 x2 + y 2
(x2 + y 2 )2
+ fxy (

x
y
2xy
y 2 x2
,
)

x2 + y 2 x2 + y 2 (x2 + y 2 )2 (x2 + y 2 )2

+ fx (

x
y

y 2 x2
,
)

(
)
x2 + y 2 x2 + y 2 x (x2 + y 2 )2
+ fyx (

x
y
y 2 x2
2xy
,
)

2
2
2
2
2
2
2
2
x +y x +y
(x + y ) (x + y 2 )2

x
y
2xy
+ fyy ( 2
,
)

(
)
x + y 2 x2 + y 2
(x2 + y 2 )2
+ fy (

x2

x
y

2xy
, 2
)
( 2
)
2
2
+y x +y
x (x + y 2 )2

x
y
y 2 x2
= fxx ( 2
,
)

(
)
x + y 2 x2 + y 2
(x2 + y 2 )2
+ 2fxy (

y
y 2 x2
2xy
x
,
)

2
2
2
2
2
2
2
2
x +y x +y
(x + y ) (x + y 2 )2
2

x
y
2xy
+ fyy ( 2
, 2
)( 2
)
2
2
x +y x +y
(x + y 2 )2
+ fx (

x
y
2x(x2 3y 2 )
,
)

x2 + y 2 x2 + y 2
(x2 + y 2 )3
+ fy (

79

x
y
2y(3x2 y 2 )
,
)

x2 + y 2 x2 + y 2
(x2 + y 2 )3

Now letting

(x, y) = (3/10, 1/10)

gives:

fxx (3/10, 1/10) = fxx (3, 10) (8)2 + 2fxy (3, 10) (8) (6) + fyy (3, 10) (6)2
+ fx (3, 10) 36 + fy (3, 10) 52
= 2 64 + 2 5 48 + (4) 36 + (8) 36 + 7 52
= 540

17.

Consider the following conditions for a dierentiable function

f (x, y) :

f (2, 1) = 8

Let

An equation for the tangent line to the level curve

(2, 1)

is

3x 5y = 1

be the tangent plane to the graph of

In each of the parts

in

z = f (x, y)

there are functions satisfying the conditions

P,
P.

(2, 1, 8) .

Determine whether

- ,
- ,

but they do not all have the same

or

there are functions satisfying the conditions


tangent plane

at the point

(a-e) below a rd condition is given.

there is no function satisfying the conditions

tangent plane

f (x, y) = 8 in the xy -plane at the point

and all of these functions have the same

(In this case nd an equation of

too.)

a.

f (3, 2) = 11

b.

fx (2, 1) = 1

c.

d
f (t2 + 1, t3 ) = 6
dt
t=1

d.

The line with parametric equations

x = 4t + 2 , y = 2t + 1 , z = t + 8 , ( < t < ) ,

e.

The line with parametric equations

x = t + 2 , y = 2t + 1 , z = t + 8 , ( < t < ) ,

perpendicular to

Solution:

is no

b.

The condition

c. c is uniquely
such c in part (e):

constant

fx (2, 1) = 3c

and

fy (2, 1) = 5c

in parts

for some

(b-d) and there

c = 1/3. Hence fy (2, 1) = 5/3. Therefore all functions


satisfying the conditions - have the same tangent plane P . An equation
for this common tangent plane P is 3x 5y + 3z = 25, and an example of a
function satisfying the given conditions is f (x, y) = x + 5/3 y + 25/3.

If

fx (2, 1) = 1

implies that

determined by the condition

gives

80

lies

is

c.

As we have

d
f (t2 + 1, t3 ) = fx (t2 + 1, t3 ) 2t + fy (t2 + 1, t3 ) 3t2
dt
t = 1 we obtain 3c2+(5c)3 = 6 and c = 2/3.
Hence fx (2, 1) = 2 and fy (2, 1) = 10/3. Therefore all functions satisfying the
conditions - have the same tangent plane P . An equation for this common
tangent plane P is 6x 10y + 3z = 26, and an example of a function satisfying
the given conditions is f (x, y) = 2x + 10/3 y + 26/3.
by the Chain Rule, substituting

d.

As the tangent plane

e.

As the tangent plane

P will have an equation of the form z = 3c(x2)+(5c)


(y 1) + 8, for the line with parametric equations x = 4t + 2 , y = 2t + 1 , z =
t + 8 , ( < t < ) , to lie in P we must have 1 = (3c) 4 + (5c) 2 and c = 1/2.
Hence fx (2, 1) = 3/2 and fy (2, 1) = 5/2. Therefore all functions satisfying
the conditions - have the same tangent plane P . An equation for this
common tangent plane P is 3x 5y 2z = 15, and an example of a function
satisfying the given conditions is f (x, y) = 3/2 x 5/2 y + 15/2.
3c (x 2) + (5c)
(y 1) (z 8) = 0, for the line with parametric equations x = t + 2 , y =
2t + 1 , z = t + 8 , ( < t < ) , to be perpendicular to P we must have the the
normal vector n = 3ci 5cj k of the plane and the velocity vector v = i +2j + k
of the line to be parallel. This requires 3c/(1) = (5c)/2 = 1/1, which is not
possible. Therefore there is no function f satisfying the conditions - .

Finally, in part

will have an equation of the form

(a), both of the functions f (x, y) = 3(x 2) 5(y 1) + 8 + 5(x 2)2

f (x, y) = 3(x 2) + 5(y 1) + 8 + (x 2)2 satisfy the conditions - , but


equations of their graphs' tangent planes P at (2, 1, 8) are z = 3x 5y + 7 and
z = 3x + 5y + 9, respectively. Therefore, in this case there are functions satisfying
the conditions - , but they do not all have the same tangent plane P .
and

18.

Let

P0 (3, 2)

f (x, y) = x3 y xy 2 + cx2

c is a constant.
A = 2i + 5j.

where

in the direction of the vector

Find

c if f

increases fastest at the point

Solution:

f = (3x2 y y 2 +2cx)i+(x3 2xy)j (f )P0 = (50+6c)i+15j . Since f


increases the fastest at P0 in the direction of (f )P0 , A = 2i + 5j must be a positive
multiple of (f )P0 . Hence (50 + 6c)/2 = 15/5 c = 22/3 . Finally we check that
c = 22/3 gives (f )P0 = 6i + 15j = 3A which is indeed a positive multiple of A.

19.

Find a vector that is tangent to the intersection curve of the surfaces

z = xy

at the point

Solution:
are level

P0 (1, 2, 2).

f (x, y, z) = x2 +y 2 +z 2
surfaces of f and g .
Let

and

g(x, y, z) = xy z .

Then the given surfaces

f = 2xi + 2yj + 2zk (f )P0 = 2i + 4j + 4k and g = yi + xj k


= 2i + j k . (f )P0 is normal to the surface dened by x2 + y 2 + z 2 = 9 and

We have

(g)P0

x2 + y 2 + z 2 = 9

81

and

(g)P0

z = xy . Therefore,
j k RRRR
R
4 4 RRRR = 8i + 10j 6k ,
R
1 1 RRRR

is normal to the surface dened by

(f )P0 (g)P0

RR i
RRR
= RRRR 2
RR
RR 2
R

or any multiple of it, is tangent to both of these surfaces, and hence, to their curve
of intersection at

20.

P0 .

In the gure below some of the level curves and the corresponding values of a nice function

f (x, y)

are shown.

a.

Draw

b.

Determine the signs of the derivatives

Remark:

f (0, 0)

as best you can on the gure.

fx , fy , fxx , fyy , fxy

at the origin.

This is an approximation problem and it should be solved under the assumption

that we are dealing with a function for which good approximations can be made using only the
given data.

82

Solution: a.
passing through

We know that

(0, 0)

f (0, 0)

is perpendicular to the level curve of

and points in the direction

increases, hence lies along the

pink vector in the gure.

We also know that the length of

f (0, 0)

is the rate of change of

with respect to

distance in this direction. On the gure using the teal circle we measure that the
pink vector crosses the level curve

f =1

0.44 units
1/0.44 2.27

at a distance of approximately

from the origin. Hence along this direction

increases at an rate of

in this direction. So now we draw a vector of this length in this direction as shown
in the next gure and this is our approximate

(The

f (0, 0).

gure is on the next page .)

83

b.

Considering the components of

f (0, 0) in part (a) it can be seen that fx (0, 0) >

0 and fy (0, 0) < 0. This can also be done in the following way: As we move along the
x-axis (the lime line in the gure on the next page) to the right, we meet level curves
belonging to higher values of f; hence fx (0, 0) > 0. Similarly, as we move along the
y -axis (the orange line) upwards, we meet level curves belonging to smaller values
of f values; hence fy (0, 0) < 0.

Moreover, note that as we move in the positive direction along both axes, the points
where we meet the level curves of

belonging to values with the same dierence

f
fyy (0, 0) > 0

become farther apart. This means that the absolute value of the rate of change of
is getting smaller. This in turn means
as

fy (0, 0) < 0.

(Part

(b)

fxx (0, 0) < 0

as

fx (0, 0) > 0,

is continued on the next page .)

84

and

Finally, we compare the rate of changes along the lime and turquoise lines.

The

level curves intersect the turquoise line at points farther apart than the points they
intersect the lime line. Hence
upwards along

Remark:

y -axis.

fx ,

which is positive, is getting smaller as we move

In other words,

fxy (0, 0) < 0.

It is also possible to nd approximate values of these derivatives using the gure. In

fact, those of

fx (0, 0)

and

fy (0, 0)

can already be read o as the components of

85

f (0, 0).

21.

Let

be a constant. Find and classify all critical points of

Solution:

f (x, y) = x3 3axy + y 3 .

fx = 3x2 3ay = 0 and fy = 3ax + 3y 2 = 0. If a = 0,


then 3x2 = 0 x = 0 and 3y 2 = 0 y = 0, and (0, 0) is the only critical point.
If a =
/ 0, then the rst equation gives y = x2 /a, and substituting this in the second
equation we get x4 a3 x = 0 whose solutions are x = 0 and x = a. Now using y = x2 /a,
we get (0, 0) and (a, a) as the critical points.
At a critical point

We compute the discriminant:

=
As

(a, a) = 27a2

fxx fxy
6x 3a
=

fyz fyy
3a 6y

fxx (a, a) = 6a, (a, a) is a local minimum for a > 0 and


a < 0. On the other hand, (0, 0) = 9a2 implies that (0, 0) is
a =/ 0.
and

local maximum for


saddle point for

a
a

(0, 0) in the case a = 0. As (0, 0) = 0,


the second derivative test fails in this case. If we restrict f (x, y) = x3 + y 3 to the
x-axis we get f (x, 0) = x3 . Since this single variable function does not have a local
Now we look at the sole critical point

maximum or minimum at
at

(0, 0)

x = 0, f (x, y) cannot have a local maximum


(0, 0) is a saddle point when a = 0.

or minimum

either. We conclude that

22.

Find the absolute maximum and minimum values of the function


on the unit disk D = {(x, y) x2 + y 2 1}.

f (x, y) = 2x3 +2xy 2 xy 2

Solution:

We rst nd the critical points of f (x, y) in the interior of D . At a


critical point we have fx = 6x2 + 2y 2 1 = 0 and fy = 4xy 2y = 0. The second
equation implies that
obtain

y = 1/ 6

y=0

or

x = 1/2.

Substituting these into the rst equation we

in the rst case, and no solution in the second case. Therefore

(x, y) = (1/ 6, 0)
interior of D .

the critical points are


points lie in the

and

(1/ 6, 0).

Note that both of these

D, that is, the unit circle x2 + y 2 = 1. We can


1. These solutions correspond to the upper
and lower semicircles.
Then f (x, 1 x2 ) = x2 + x 1 for 1 x 1, and

d
1
f (x, 1 x2 ) = 2x + 1 = 0 x = . This gives the critical points (x, y) =
dx
2

(1/2, 3/2) and (1/2, 3/2) of the restriction of f to the boundary of D. We


must also include the endpoints x = 1 and x = 1, in other words, the points
(x, y) = (1, 0) and (1, 0) in our list.
Now we look at the boundary of

solve y as y = 1 x2 , 1 x

Hence the absolute maximum and the absolute minimum of


of the points

on

occur at some

(1/ 6, 0), (1/ 6, 0), (1/2, 3/2), (1/2, 3/2), (1, 0), (1, 0) .

The values of

at these points are

2 1
,
3 3

2
5
5
, , , 1 , 1 ,
3
4
4
86

respectively. Therefore the absolute maximum value is 1 and the absolute minimum
value is

Remark:

5/4.

Here are two more ways of dealing with the critical points of the restriction of

the boundary of

to

D.

In the rst one we parametrize the boundary, which is the unit circle, by

< t < .

x = cos t, y = sin t,

Then

d
d
1
f (cos t, sin t) = (cos2 t + cos t 1) = 2 cos t sin t sin t = 0 cos t =
dt
dt
2

and these give us (x, y) = (1/2,


3/2), (1/2, 3/2), (1, 0), (1, 0).

or

sin t = 0 ,

g(x,y)

In the second we use the Lagrange Multipliers Method for the boundary

O2


x2 + y 2 = 1.

f = gx
6x2 + 2y 2 1 = 2x

f = g
4xy 2y = 2y
} fy = gy
g = 1

g
=
1
x2 + y 2 = 1

O1
O2
O3

gives y = 0 or x = (1 +)/2. If y = 0, then 3 gives x = 1 (and = 5/2). On the other hand,

2 2x + 1 = 0 x = 1/2 and y = 3/2. Therefore


x = (1 + )/2, then from 1 and 3 , 4x
3/2), (1/2, 3/2), (1, 0), (1, 0) are added to the list.
the points (x, y) = (1/2,
if

87

23.

Find the absolute maximum and minimum values of the function

f (x, y) = 2(x2 + y 2 1)2 + x2 y 2


on the unit disk

Solution:

D = {(x, y) x2 + y 2 1} .
We rst nd the critical points of

fx = 4(x2 + y 2 1) 2x + 2x = 0

and

in the interior of

D.

We want:

fy = 4(x2 + y 2 1) 2y 2y = 0 .

The rst equation gives x = 0 or


or 4x2 + 4y 2 5 = 0. Therefore x

4x2 + 4y 2 3 = 0, and the second one gives y = 0


= 0 and y = 0, or 4x2 + 4y 2 3 = 0 and y = 0, or
x = 0 and 4x2 + 4y 2 5 = 0, or 4x2 + 4y 2 3 = 0 and 4x2 + 4y 2 5 = 0. In the last
case
there is no solution, and the rst three cases leads to the points (0, 0), ( 3/2, 0)

and (0, 5/2). Only the rst three of these critical points lie in the interior of D
as for the last two x2 + y 2 = 5/4 > 1.
Next we consider the restriction of f to the boundary of D which is the unit circle.

As on the boundary y = 1 x2 , we consider the functions f (x, 1 x2 ) = 2x2 1

on the interval 1 x 1. As (d/dx)f (x, 1 x2 ) = 4x, the only critical point of


both of these restrictions occur when

(0, 1).

x = 0.

On the other hand, for both

the interval

1 x 1

give the points

Now we nd the values of

(x, y) = (0, 1) and


functions the endpoints x = 1 and x = 1 of
(x, y) = (1, 0) and (1, 0).
This leads the points

at these seven points:

f (0, 0) = 2

f ( 3/2, 0) = f ( 3/2) = 7/8


f (0, 1) = f (0, 1) = 1
f (1, 0) = f (1, 0) = 1
Therefore the absolute maximum and minimum values of
respectively.

88

on

are

and

1,

Remark:

The boundary can also be dealt with using the parametrization x = cos t, y = sin t,
< t < ), of the unit circle. Then the restriction of f to the unit circle gives the function
f (cos t, sin t) = cos 2t on the interval < t < . (d/dt)f (cos t, sin t) = 2 sin 2t = 0 means that
t is an integer multiple of /2 and this gives the critical points (x, y) = (1, 0) and (0, 1) for
the restriction of f to the boundary.
(

Remark:

Consider the function g(x, y) = 2(x2 + y 2 1)2 + 2xy on the unit disk D . This is the
same function as f rotated 45 counterclockwise. Hence the absolute maximum value of g on

(x, y) = (0, 0)
the absolute minimum value 1 at the
, and
(x, y) = (1/ 2, 1/ 2). It is a good exercise to solve this

will be 2 assumed at the point

points

(x, y) = (1/ 2, 1/ 2)

and

problem directly.

24.

Three hemispheres with radiuses

1, x and y , where 1 x y 0, are stacked on top of each


h.

other as shown in the gure. Find the largest possible value of the total height

89

Solution:

We want to maximize

h(x, y) =

1 x2 +

x2 y 2 + y

on the closed and bounded region

D = {(x, y) 0 y x 1}.

We rst nd the critical points of

hx =

x
1 x2

x
x2 y 2

From the second equation we obtain


used the fact that x > 0 and
12y 2 = 2y 2 y 2 3y 2 = 1
critical point of

is

in the interior of

=0

and

D.

hy =

We want

y
x2 y 2

+ 1 = 0.

y 2 = x2 y 2 x2 = 2y 2 x =

2y where we

y > 0. Nowsubstituting this in the rst


equation gives

. So the only
y = 1/ 3 as y > 0 and hence x = 2/3

(x, y) = ( 2/3, 1/ 3) and it lies in D as 0 1/ 3 2/3 1.

Now we consider the restriction of

to the boundary of

D.

y =0
1 x2 + x

Side 1 : On the bottom edge of the triangle, we


have

therefore we are considering the function

h(x, 0) =

0 x 1,
0 x 1.

and
for

and

d
x
1
h(x, 0) =
+ 1 = 0 x2 = 1 x2 2x2 = 1 x =
dx
2
1 x2
x > 0. Taking
the endpoints x = 0
points (x, y) = (1/ 2, 0), (0, 0), (1, 0).
as

and

x=1

into account, Side 1 gives us the

Side 2 : On the right edge of the triangle,


we have x = 1 and
h(1, y) = 1 y 2 + y for 0

0 y 1, and therefore
y
1. As in the case of

(x, y) = (1, 1/ 2), (1, 0), (1, 1).

we are considering the function

Side 1 this leads to the points

Side 3 : On the top edge of the triangle, we


have y = x and 0 x 1, and therefore
we are considering the function h(x, x) =
1 x2 + x for 0 x 1. Once again as in
the case of Side 1 we obtain the points

(x, y) = (1/ 2, 1/ 2), (0, 0), (1, 1).

90

Now we nd the values of

at these seven points:

h(0, 0) = h(1, 0) = h(1, 1) = 1

h(1/ 2, 0) = h(1, 1/ 2) = f (1/ 2, 1/ 2) = 2

h( 2/3, 1/ 3) = 3
Therefore the maximum possible total height of the three hemispheres is

Remark:

3.

Here is a single variable argument which solves the problem for any number

hemispheres: Suppose the maximum possible height for

hemispheres is

for some

k+1

k 1.

hemispheres, where the second one from the bottom has radius r , the maximum

possible height will be H(r) =


kr + 1 r2 for 0 r 1. The value of H(r) is 1 and k at

Then for

r = 0 and r = 1, respectively; and its value is


r = k/(k + 1). Therefore we conclude inductively that the
hemispheres is
n for n 1.
the
endpoints

25.

Find the absolute maximum and the absolute minimum of


sphere x2 + y 2 + z 2 = 1.

Solution:
z 2 1.

and

z = 0,

O3

we obtain

then

the points

O3

gives

(1, 0, 0).

fx
fy
fz
g

=
=
=
=

z = 42 z ,
y = 0,

gx
gy
gz
0

3x2

z

y

2 + y2 + z2

and hence

and then

O4

z=0
gives

or

= 1/2

x = 1.

O O
O
O
O1 O3 3x = x y= z

O1
O2
O3
O4

=
=
=
=

2x
2y
2z
1

or

= 1/2.

In this case we have

= 1/2, then from 1 and 3 , 3x2 = x and y = z . Therefore we either


have x = 0 in which case y = z = 1/ 2 by 4 , or we have x = 1/3 and then

y = z = 2/3 again by 4 . In this case the critical points are (0, 1/ 2, 1/ 2)


and (1/3, 2/3, 2/3).
If

If

= 1/2,

then from

and

and

. A reasoning similar to

(0, 1/ 2, 1/ 2) and (1/3, 2/3, 2/3).

Hence the critical points are (1, 0, 0), (0, 1/ 2, 1/ 2), (1/3, 2/3, 2/3),

(0, 1/ 2, 1/ 2), (1/3, 2/3, 2/3) , and the values of f at these points are
1, 1/2, 1/2, 13/27, 13/27 , respectively. Therefore the absolute maximum is 1 and
the absolute minimum is 1.
the previous case gives the points

91

on the unit

g(x, y, z) = x2 + y 2 +

Then

O2

If

at its only critical point

f (x, y, z) = x3 + yz

We will use the Lagrange Multipliers Method. Let

f = g
}
g = 0

From

k+1

maximum possible height for

26.

Evaluate the following integrals:

a.

3
0 y sin(x ) dx dy

b.

2 x
R y e dA
2

2yy 2

c.

0 0

d.

where

R = {(x, y) 0 y x}

xy
dx dy
+ y2

x2

(x2

dy dx
+ y 2 )2 + 1

Solution: a.

We will rst express the iterated integral as a double integral and

x-integral goes from x = y to x = 1 as


gure in the xy -plane. Then the y -integral

then reverse the order of integration. The


shown by the red line segments in the
goes from

y=0

to

y = 1.

Therefore the intervals of the x-integral trace out the region R bounded by the
parabola y = x2 , the line x = 1, and the x-axis. Note that the x-integrals are always
from left to right in the interval

0 y 1.

Hence we have

3
3
0 y sin(x ) dx dy = R sin(x ) dA .

y -integral rst.
integration for the y -

Now we express this double integral as an iterated integral with the


The green line segment in the gure shows the interval of
integral which goes from y = 0 to y = x2 . Then:

92

3
3
0 y sin(x ) dx dy = R sin(x ) dA

=
=
=

x2

0
1
0
1
0

sin(x3 ) dy dx
y=x2

sin(x3 ) y]y=0 dx
sin(x3 )x2 dx

1
1
cos(x3 )]0
3
2
=
3

b.

We integrate with respect to

rst to obtain

2 x
2 x
R y e dA = 0 0 y e dy dx
1
2
= x3 ex dx
3 0
1
= tet dt
6 0
1
=
6
2

where we used the integration by parts

tet dt =

t d(et )

= lim ([tet ]c0 +


c

et dt)

c
= lim c lim [et ]c0
c e
c
1

= lim c lim (ec 1)


c e
c
=1
L'H

in the last step.

c.

This time we will use the polar coordinates. To do so we rst determine the

region of integration

R.

x= 0 to x = 2y y 2 as shown
2y y 2 x2 = 2y y 2
by the red line segment in the gure. Since x =

x2 + (y 1)2 = 12 , x = 2y y 2 gives the right semicircle of the circle x2 + (y 1)2 = 1.


On the other hand, the y -integral goes from y = 0 to y = 2. Therefore R is right half
of the disk x2 +(y 1)2 1. Note that the polar equation of the circle x2 +(y 1)2 = 1
is r = 2 sin , and to obtain the right semicircle we vary from = 0 to = /2.
In the iterated integral the

x-integral

goes from

93

Hence:

2yy 2

0 0

xy
xy
dx dy = 2
dA
2
+y
R x + y2
/2
2 sin r cos r sin
=
r dr d

r2
0
0

x2

=
=
=

/2

2 sin

0
/2
0
/2

sin cos r dr d

sin cos [

r2 r=2 sin
]
dr d
2 r=0

2 sin3 cos d

0
/2

sin4
=
]
2 0
1
=
2

d.

Again we use the polar coordinates. This time the integration region is

{(x, y) x 0 and y 0},


0

that is, the rst quadrant.

dy dx
1
=
dA

(x2 + y 2 )2 + 1
R (x2 + y 2 )2 + 1
/2
r dr d
=

r4 + 1
0
0

94

R=

/2

1
r=c
lim [arctan(r2 )]r=0 d
c
2
0
/2
=
d
4
0
2
=
8
=

27.

Evaluate the double integral

Solution:
the line

Let

y=x

1
R (x2 + y 2 )2 dA

where

R is the region shown in the gure.

be the portion of the region lying in the rst quadrant between

and the

x-axis.

By symmetry we have:

1
1
R (x2 + y 2 )2 dA = 8 R (x2 + y 2 )2 dA
/4
2 sec
1
= 8
r dr d

(r2 )2
0
sec
/4
1 r=2 sec
= 8
[ 2 ]
d
2r r=sec
0
= 3

/4

cos2 d

1 + cos 2
d
2
0
3
sin 2 /4
= [ +
]
2
2 0
3 3
=
+
8 4
= 3

/4

95

28.

Evaluate the following integrals.

a.

2
5
R x y cos(y /2) dA

where

R1

is the region shown in the gure.

2
5
R x y cos(y /2) dA

where

R2

is the region shown in the gure.

b.

y=x

y
y=1

R1

R2

x
y=x

x=1

y = x

y = x

Solution: a.

respect to the

the function

reection with respect to

Note that the region R1 is symmetric with


f (x, y) = x2 y cos(y 5 /2) changes sign under

x-axis: f (x, y) = f (x, y).

the

Therefore

x-axis,

but

2
5
R x y cos(y /2) dA = 0.
1

b.

We integrate with respect to

x,

then

y:

x=y
1 3
5
[
x
y
cos(y
/2)]
dy
y
3
0
0
x=y
1
2 1
2 2
4
= y 4 cos(y 5 /2) dy = [ sin(y 5 /2)] =
3 0
3 5
15
0

x2 y cos(y 5 /2) dA =

x2 y cos(y 5 /2) dx dy =

96

29.

Evaluate the following integral

x 1 and 0 y 1}.

Solution:

R (1 + x y) dA

where

R = {(x, y) x y 2/3 and 0

We rst observe that

R (1 + x y) dA = R dA + R (x y) dA
R is
y = x whereas the function f (x, y) = xy changes
respect to this line: f (y, x) = f (x, y).

and then note that the second integral on the right is zero because the region
symmetric with respect to the line
sign under the reection with

On the other hand the rst on the right is just the area of
Hence:

R, that is 12 (1/3)2 = 8/9.

8
8
R (1 + x y) dA = R dA + R (x y) dA = 9 + 0 = 9

97

30.

When pirates retire, they live on the Square Island

S = {(x, y) x 5
in the Sea of

and

y 5}

xy -plane where all distances are measured in hectometers (=hm).

Having lived all

their lives on it, the retired pirates want to be as far away from the sea as possible. As a result,
the pirate population density

p(x, y)

at a point

(x, y)

on the Square Island is proportional to


the distance of the point from the shore and reaches its largest value of 15 pirate/hm2 at the
center of the island. Find the total number

of pirates living on the Square Island.

Solution:

Remark:

Compare this example with

Example 44 in Part 1.

98

31.

r2 = 2 cos(2)
in the plane is the base of a solid right

2 r2 . Find the cylinder's volume.


sphere z =

The region enclosed by the lemniscate

cylinder whose top is bounded by the

Solution:

Let

R be the the region enclosed by the lemniscate in the rst quadrant.

By symmetry the volume is

2 r2 dA

= 4

2 cos 2

/4
0

2 r2 r dr d

r= 2 cos 2
1
4 /4 3/2
[ (2 r2 )3/2 ]
(2 (2 2 cos 2)3/2 ) d
= 4
d =
3
3 0
0
r=0

/4
8 2
2 2 32 /4 3
(1 (2 sin2 )3/2 ) d =
=

sin d

3
3 3 0
0

2 2 32 /4
2 2 32 1
2
=

(1 cos2 ) sin d =

(1 u ) du

3
3 0
3
3 1/ 2

1
3
2 2 32
u
2 2 32 2
5
=

[u ] =

( )
3
3
3 1/ 2
3
3 3 6 2

2 2 64 40 2
=

+
.
3
9
9
/4

99

32.

Let

D be the region in space bounded by


y = x2 on the sides, and the xy -plane

the plane

y+z = 1

on the top, the parabolic

V of the
region D in terms of iterated integrals with orders of integration (a) dz dy dx and (b) dx dy dz .
cylinder

at the bottom. Express the volume

Solution:

100

1
D (x2 + y 2 + z 2 )2 dV where D is the region bounded by the

cylinder x2 + y 2 = 1 on the sides and by the hemisphere z =


4 x2 y 2 at the bottom. Express

33.

Consider the triple integral

this integral in terms of iterated integrals in

a.

Cartesian coordinates,

b.

cylindrical coordinates,

c.

spherical coordinates, and

d.

evaluate the integral in the coordinate system of your choice.

Solution:
x2 + y 2

We observe that the projection of

1.

to the

xy -plane

is the unit disk

A vertical line passing through a point of the unit disk enters the region

at a point on the hemisphere


and remains in the region from there on. The equations

of the hemisphere z =
4 x2 y 2 and the cylinder x2 + y 2 = 1 are z = 4 r2 and

r = 1,

respectively, in cylindrical coordinates. The answers to parts

(a),

1x2

1
1
1
dV
=

D (x2 + y 2 + z 2 )2
1 1x2 4x2 y2 (x2 + y 2 + z 2 )2 dz dy dx ,
and

(b),
2
1

1
1
D (x2 + y 2 + z 2 )2 dV = 0 0 4r2 (r2 + z 2 )2 r dz dr d ,

immediately follow from these observations.


To do part

(c) we further observe that

the equations of the hemisphere and the cylinder are

=2

and

sin = 1 ,

respectively,
a ray starting at the origin enters the region at a point on the hemisphere and
leaves the region at a point on the cylinder, and
such a ray intersects the region exactly when
through a point on the intersection circle when

0 /6.
= /6.)

(The ray passes

Therefore,

2
/6
csc 1
1
2 sin d d d .
D (x2 + y 2 + z 2 )2 dV = 0 0 2
4

101

We use this last iterated integral for the computation

(d):

2
/6
csc 1
1
sin d d d
D (x2 + y 2 + z 2 )2 dV = 0 0 2
2
2
/6
1 =csc
=
[
]
sin d d
0
=2
0
2
/6
1
=
( sin + ) sin d d

2
0
0
2
/6
1 cos 2 sin
=
+
) d d
0 (
2
2
0
2
sin 2 cos =/6
=
[ +

]
d
2
4
2 =0
0

2
3
3 1

+ ) d
=
( +
12
8
4
2
0

3
= (1
)
6
4

102

34.

be the region in space bounded on the top by the sphere x2 + y 2 + z 2 = 2 and on the
bottom by the paraboloid z = x2 + y 2 . Express the volume V of D in terms of iterated integrals
Let

in the

(a) Cartesian, (b) cylindrical, and (c) spherical coordinates.

Solution:

Note that the projection of the curve of intersection of the sphere and
x2 + y 2 = 1 in the xy -plane. This curve also bounds

the paraboloid is the unit circle


the projection of the solid

V =

to the

Hence we have

2x2 y 2

1x2

1
1

xy -plane.

1x2 x2 +y2

and

V =
0

dz dy dx

2r2

0 r 2

r dz dr d .

D through the sphere if the


angle it makes with the positive z -axis is less than /4, whereas it does so through
the paraboloid if this angle is between /4 and /2. Hence:

Now observe that a ray starting at the origin leaves

V =
0

/4

2 sin d d d

2
0

/2

/4 0

103

cos / sin2

2 sin d d d

35.

Consider the iterated integral

/2

2r2

0 r

dz dr d

in cylindrical coordinates.

a.

Change the order of integration into

b.

Express the integral in spherical coordinates with order of integration

Solution: a.
z = 2

x2

y2.

z = r

is the cone

z 2 = x2 + y 2 ,

and

z = 2 r2

d d d .

is the paraboloid

These surfaces intersect along a circle that is also the curve of

intersection of the cylinder

0 /2,

dr dz d.

r=1

and the horizontal plane

the integration region

z = 1.

Since

0r1

and

is the region in the rst octant bounded by

the paraboloid on the top and the cone at the bottom.

When we integrate with respect to

rst, we will be moving along a curve on

104

which

and

are constant. These two conditions describe a horizontal plane and a

z -axis, respectively. Therefore the r-integration is along a ray


perpendicular to the z -axis like the blue ones in the gure. Such a ray starts on the
z -axis in D, and leaves D through the cone if 0 z 1, and through the paraboloid
if 1 z 2. Therefore,
plane containing the

/2

2r2

0 r

dz dr d =

/2

0 0 dr dz d

+
where we used the fact that

b.

We have

/2

0
as

z = 2 r2

dV = r dz dr d

(a).

and

2r2

0 r

and

r 0 r =

in cylindrical coordinates where

2z

1 0

dz dr d =

Since we want to integrate with respect to


on which

/2

dr dz d

2z.

1
dV
r

is the region described in part

rst, we will be moving along the curves

are constant. These two conditions describe a sphere with center

z -axis, respectively. Therefore the


-integration takes place along a vertical semicircle subtended by a diameter along
the z -axis and with center at the origin like the green ones in the gure. Such a
semicircle starts on the positive z -axis in D , and leave the region of integration

intersecting the cone for 0


2 and the paraboloid for 2 2. The upper
half of the cone z = r has the equation = /4 in spherical coordinates. On the

2
2
other hand, z = 2 r cos = 2 ( sin ) cos = (1 +
42 7)/(2) for
a point on the paraboloid as /4 . Therefore,

at the origin and a half-plane whose spine is the

1
1
D r dV = D sin dV
=

/2
0

/4

0
+

where we substituted

d d d
/2

2 0

dV = 2 sin d d d

coordinates.

105

arccos((1+ 42 7)/(2))

d d d
for the volume element in spherical

36.

The region

about the

z -axis

to obtain a solid

iterated integrals in

Solution:

z 2 = y 2 y 4 in the right half of the yz -plane is rotated


the xyz -space. Express the volume V of D in terms of

bounded by the curve

in

(a) the Cartesian, (b) the cylindrical and (c) the spherical coordinates.

Since the surface bounding the the solid

is obtained by revolving a

z -axis, its equation in the cylindrical coordinates will not depend


on . Since r = y in the yz -plane and the curve has the equation z 2 = y 2 y 4 , we
conclude that z 2 = r 2 r 4 is the equation of the surface in the cylindrical coordinates.
curve about the

From this

V =
0
follows as the projection of

r2 r4

0 r2 r4 r dz dr d

to the

xy -plane

is the unit disk.

The equation z 2 = r 2 r 4 in the cylindrical coordinates transforms to z 2 = (x2 +


y 2 ) (x2 + y 2 )2 in the Cartesian coordinates, and once again using the fact that the
projection of

to the

xy -plane

is the unit disk, one now obtains:

1x2

(x2 +y 2 )(x2 +y 2 )2

V =
(x2 +y2 )(x2 +y2 )2 dz dy dx
1 1x2
The spherical coordinates require a little bit more work. First note that the equation
z 2 = r2
r4 in the cylindrical coordinates now gives ( cos )2 = ( sin )2 ( sin )4 ,
or =
sin2 cos2 / sin2 , in the spherical coordinates. Next note that for
2
sin cos2 = cos 2 to be nonnegative, must be between /4 and 3/4 in the
interval

[0, ].

Therefore:

V =
0

Remark:

sin2 cos2 / sin2

3/4

/4

Compare this example with

2 sin d d d

Example 48 in Part 1.

106

37.

0
Evaluate the iterated integral

2y+3

1 y

x+y
ex2y dx dy .
2
(x 2y)

Solution:

38.

Let

(u, v)

and

Solution:

(x, y)

be two coordinate systems. Show that

By denition,

(x, y) xu xv
= xu yv xv yu
=
(u, v) yu yv
and

(u, v) ux uy
= ux vy uy vx .
=
(x, y) vx vy
107

(x, y) (u, v)

= 1.
(u, v) (x, y)

Therefore,

(x, y) (u, v)

= (xu yv xv yu )(ux vy uy vx )
(u, v) (x, y)
= xu ux yv vy + xv vx yu uy xu uy yv vx xv ux yu vy
= xu ux yv vy + xv vx yu uy xu uy yv vx xv ux yu vy
+ xv vx yv vy + xu ux yu uy xv vx yv vy xu ux yu uy
= (xu ux + xv vx ) (yu uy + yv vy )
(xu uy + xv vy ) (yu ux + yv vx )
= xx yy xy yx
=1100
=1

39.

Evaluate the double integral

Solution:

R e

x2 /y

dA

where

R = {(x, y) x2 y

x }.

u = y 2 /x, v = x2 /y . In this coordinate


G = {(u, v) 0 u 1 and 0 v 1} .

We change the coordinates to

system the region of integration becomes

We have

1
(u, v) ux uy
(x, y)
1
y 2 /x2 2y/x
=
= ,
=
= 3
=
2
2
2x/y x /y
(x, y) vx vy
(u, v) (u, v)
3
(x, y)
and the change of variables formula gives

x2 /y

R e

dx dy = ev
G

1
1
1
e1
(x, y)
du dv = ev dv du =
.
(u, v)
3
3
0
0

108

Remark:
any

We can multiply dierentials. The rules are

uv -coordinate

du du = 0 = dv dv

and

dv du = du dv

in

system. For instance, when changing from Cartesian to polar coordinates,

we have

dx dy = d(r cos ) d(r sin )


= (cos dr r sin d)(sin dr + r cos d)
= r sin2 d dr + r cos2 dr d
= r(sin2 + cos2 ) dr d
= r dr d
and in

Example 39 we have
du dv = d(y 2 /x) d(x2 /y)
= (2y/x dy y 2 /x2 dx)(2x/y dx x2 /y 2 dy)
= 4 dy dx + dx dy
= 4 dx dy + dx dy
= 3 dx dy ,

hence

1
dx dy = du dv .
3

This can be used to keep track of how the area element changes under

a coordinate change, but note that the sign of the factor in front must be corrected by hand so
that it is positive on the region of integration.

40.
C

Consider the transformation

T x=

such that the inequality

u
v
,y=
u+v+1
u+v+1

. Show that there is a constant

(x, y)
G (u, v) du dv C
holds for all regions

contained in the rst quadrant of the

uv -plane.

u
v
u+v
0, y =
0 and x + y =
1
u+v+1
u+v+1
u+v+1
x
y
for u 0 and v 0. We also have u =
and v =
. Therefore T maps
1xy
1xy
the rst quadrant of the uv -plane into the triangle R = {(x, y) x+y 1, x 0, y 0}

Solution:

Observe that

x=

in a one-to-one manner.

109

Hence:

(x, y)
G (u, v) du dv = T (G) dx dy = (Area

Remark:

of

T (G)) (Area

of

R) =

1
2

One can also do a straightforward computation.

1
(x, y) xu xv
(v + 1)/(u + v + 1)2
u/(u + v + 1)2
=
=
=
2
2
v/(u + v + 1)
(u + 1)/(u + v + 1)
(u, v) yu yv
(u + v + 1)3
and hence

du dv
(x, y)
1 dv
1

du
dv

=
G (u, v)
0 0 (u + v + 1)3 2 0 (v + 1)2 = 2 .

41.

Compute the Jacobian

(x, y, z)
(, , )

where

(, , ) are the spherical coordinates and (x, y, z)

are the Cartesian coordinates.

Solution:

We have

x = sin cos , y = sin sin , z = cos .

Therefore

RRx x x RR
(x, y, z) RRRR RRRR
= RR y y y RRR
RR
(, , ) RRRR
RR z z z RRR
RRRsin cos cos cos sin sin RRR
RR
RR
= RRRR sin sin cos sin sin cos RRRR
RRR
RRR
sin
0
RR cos
RR
sin sin cos sin
= ( sin sin )

cos
sin
( sin cos )

sin cos cos cos

cos
sin
+0

sin cos cos cos

sin sin cos sin

= ( sin sin )( sin )(sin2 + cos2 )


( sin cos )( cos )(sin2 + cos2 )
+0
2
2
2
= sin (sin + cos )
= 2 sin
where we used the cofactor expansion with respect to the third column.

110

Part 3: Sequences and Series

1. Let a1 = 1, a2 = a3 = 2, a4 = a5 = a6 = 3, a7 = a8 = a9 = a10 = 4,
an 1, 2, 2, 3, 3, 3, 4, 4, 4, 4, 5, 5, 5, 5, 5, 6, . . . . What is a2015 ?

and so on.

That is,

Solution:

an = k if 1 + 2 + + (k 1) < n 1 + 2 + + k . In other words, an = k if


k(k 1)/2 < n k(k + 1)/2. Since for k = 63, k(k 1)/2 = 1953 and k(k + 1)/2 = 2016,
we have a2015 = 63.

Remark:
2.

A more explicit formula

8n + 1 1
an =

can be obtained.

Curve 1 is an equilateral triangle with unit sides. For

n 2, Curve n is obtained from Curve n1

by replacing the middle third of every edge with the other two sides of the outward pointing

Ln be the
Ln , An , lim Ln

equilateral triangle sitting on it. Let

length of Curve n and

region enclosed by Curve n . Find

and

Solution:

Let

en

and

dn

An

be the area of the

lim An .

denote the number of edges and the length of each edge

en = 4en1 and dn = dn1 /3 for n 2, and e1 = 3


and d1 = 1, we nd that en = 3 4n1 and dn = 1/3n1 for n 1. It follows that
Ln = en dn = 3 (4/3)n1 for n 1, and lim Ln = lim 3 (4/3)n1 = .
of Curve n , respectively.

Since

On the other hand, the

region is obtained by adjoining

en1

equilateral triangles

(n 1) region. Therefore,

3 (dn1 /3)2
3 1 2
An = An1 + en1
= An1 + 3 4n2
(
)
4
4 3n1

of side length

dn1 /3

nth

and then

to the

st

1
4 n2
4 n3
An = (( ) + ( ) + + 1) + A1
9
4 3 9

3
A1 =
, this gives
4

3 3
4 n1
3
An =
(1 ( ) ) +
20
9
4

for

n 2.

Since

for

n 1.

We obtain

3 3
3 2 3
lim An =
+
=
.
n
20
4
5

111

Remark:

Curve 1 :

Curve 2 :

Curve 3 :

Curve 4 :

t = 0 sec, takes out 1 ball at t = 1/2 sec,


takes out 1 ball at t = 7/8 sec, puts 3 balls at t = 15/16 sec, takes out

A magician puts 1 ball into an empty box at

t = 3/4 sec,
t = 31/32 sec, and
box at t = 1 sec.

puts 2 balls at
1 ball at
in the

so on. Then she challenges you to guess how many balls there are

You ask for the advice of your friends.

Friend1 says: The net result of

2k + 1st

and

2k + 2nd

steps for

k1

is to put at

least one more ball into the box. There are innitely many balls in the box at

t=1

sec.

ball1, ball2, ball3, and so on


ball1 at t = 0
sec, takes out ball1 at t = 1/2 sec, puts ball2 and ball3 at t = 3/4 sec, takes out
ball2 at t = 7/8 sec, puts ball4, ball5, and ball6 at t = 15/16 sec, takes out ball3
at t = 31/32 sec, and so on. For any given k , ballk is not in the box at time t = 1
Friend2 says: Imagine that the balls are labeled as

with invisible ink which only the magician can see. Then she puts

sec, because it was taken out at time


box at time

t=1

t = 1 1/22k1

sec.

What do you think?

112

sec. There are no balls in the

3.

Let the sequence

{an }

be dened by

a1 = 1

and

an =

sequence converges and nd its limit.

Solution:

for

n 1.

Show that the

0 < 1 + an < 1 + an+1 and an+1 =


1/(1 + an ) > 1/(1 + an+1 ) = an+2 . Similarly, if an > an+1 > 0, then 1 + an > 1 + an+1 > 0
and an+1 = 1/(1 + an ) < 1/(1 + an+1 ) = an+2 . Since a1 = 1 > 1/2 = a2 , it follows that
Observe that if

0 < an < an+1 ,

1
1 + an1

then

1 = a1 > a3 > a5 > > a6 > a4 > a2 =

1
.
2

Therefore, the sequence {a2n }n=1 is increasing and bounded from above by 1, and

the sequence {a2n1 }n=1 is decreasing and bounded from below by 1/2. By the
Monotonic Sequence Theorem, then both of these sequences are convergent.

lim a2n1 = L and lim a2n = M . Taking the limit of a2n+1 = 1/(1 + a2n ) as n
n
n
we obtain L = 1/(1 + M ), and taking the limit of a2n = 1/(1 + a2n1 ) as n we
Let

From M + M L = 1 and LM +

Therefore the sequence {an }n=1 converges to L = M .


obtain

M = 1/(1 + L).

L=1

it follows that

M = L.

L2 + L 1 = 0 gives L = ( 5 1)/2
or L = ( 5 + 1)/2. Since 0 < an
n 1, we must have 0 L . Hence L = ( 5 1)/2 is the limit and

51
lim an =
.
n
2

Finally,

Remark:

This result is sometimes expressed symbolically in the form:

51
=
2

1
1

1+

1+

1+

1+
1+

Remark:

1
1+

xn = Fn /Fn+1 for n 1 where Fn 1, 1, 2, 3, 5, 8, . . .


showed that lim Fn /Fn+1 = ( 5 1)/2 .

Note that

sequence. So we

Remark:

for all

is the Fibonacci

It can be showed with a little bit more work that any sequence satisfying the given

(5 1)/2 if a1 =/ (1 + 5)/2 and a1 =/ Fn+1 /Fn for any n 1 .


5)/2 , then the sequence is constant; and if a1 = Fn+1 /Fn for
On the other hand, if a1 = (1 +
some n 1, then an = 1 and an+1 is undened.
recursion relation converges to

113

4.

Let the sequence

{xn }

be dened by

x0 = 2

and

xn =

xn1
1
+
2
xn1

a.

Find the limit of this sequence assuming it exists.

b.

Show that the limit exists.

Solution: a.
xn =

We assume that the limit

xn1
1
+
2
xn1

L = lim xn

n 1 xn xn1 =

for

for

n 1.

exists. Then

x2n1
+1
2

for

n1

x2n1
L2
+ 1 L2 =
+ 1 L2 = 2
2
2

L = 2 or L = 2.

lim(xn xn1 ) = lim


as

lim xn1 = lim xn = L .

Therefore

x0 = 2 > 0 and if xn > 0 then xn+1 = xn /2 + 1/xn > 0 + 0 = 0. Hence by induction


xn > 0
for all n 0. It follows that L = lim xn 0. We conclude that the limit is
2.

b.

In part

(a) we proved that xn > 0 for all n 0.

Therefore

{xn } is bounded from

below.
Now we will show by induction on

Let

n = 0.

Let

n>0

that

2 < xn+1 < xn

x0 = 2, x1 = 3/2, and x21 = 9/4 > 2,

assume that
2 < xn+1 < xn . Then

Since
and

xn+1 xn+2 =
and hence

xn+2 < xn+1 .

and since

xn+2 > 0,

we have

n 0.

2 < x1 < x0 .

x2 2
xn+1
1

= n+1
>0
2
xn+1
2xn+1

On the other hand,

x2n+2

for all

x2 2
xn+1
1
2=(
+
) 2 = ( n+1
) >0
2
xn+1
2xn+1

we have

xn+2 >

2.

Therefore the sequence is decreasing.


Since the sequence is bounded from below and decreasing, we conclude that it
converges by the Monotonic Sequence Theorem.

114

5.

We have a

1/n1/n

n. For each of (a-c), determine whether


xy -plane in such a way that they completely

square for each positive integer

it is possible or not to place these squares in the


cover the given set.

1
1/2

1/3
1/2

1/4

a.

The entire

b.

The line dened by the equation

y=x

c.

The region between the graph of

y = ex

xy -plane

Solution: a.
as it is the

...

1/4
1/3

and the

x-axis

for

x0

1
. This series is convergent
2
n=1 n
sum is 2 /6.) On the other hand,

The sum of the areas of the squares is

p-series

with

p = 2 > 1.

(In fact, its

the entire plane has innite area. Hence it is not possible to cover the entire plane
using these squares.

b.

We place the odd-numbered squares along the half of the line lying in the rst

quadrant and the even-numbered ones along the half lying in the third quadrant

as shown in the gure. As

2
1
1
= =
2k
2 k=1 k

and

k=1
entire line is covered by the squares.

c.

We place the squares along the positive

the

x-axis

squares extend along the entire positive axis as

115

2
2
>
= ,
k=1 2k 1
k=1 2k

as shown in the gure. Then the

1
= .
n=1 n

On the other hand, by

the Integral Test Inequality we have

1+
for all

n 1.

n+1 dx
1 1
1
+ ++ >
= ln(n + 1)
2 3
n
x
1

(This inequality can be obtained by comparing the area covered by

rectangles erected on the unit intervals along the interval


corners on the graph of

y = 1/x

Hence

ex e(1+1/2+1/3++1/n)
n

for

n+1
1
1
x
k=1 k
k=1 k

for all

n 0.

[1, n + 1]

with upper left

and the area under the graph on this interval.)

1
n+1

In other words, the top side of the

nth

rectangle

never lies lower than the graph of y = ex . Therefore we can cover the region between
the graph of y = ex and the positive x-axis with the squares.

6.

For each of the series in

the

nth

partial sum

a.

sn

(a-d), determine whether there exists a positive integer n such that

of the series satises the condition

1
n
n=1 2

Solution:

b.

c.

n=1

2014 sn 2015.
n

2999
)
(
n=1 3000

d.

1
n=1 n

Note that all terms of these series are positive, and therefore, their partial

sums form increasing sequences. This will be used repeatedly in the following.

a.

We have

1/2
1
1
<
=
= 1 < 2014

k
n
1 1/2
n=1 2
k=1 2
n

sn =
for all

n 1,

where we used the geometric series sum formula, and hence there are

no partial sums lying between 2014 and 2015.

b.

This time we have

k=1

k=1

sn = 5k 5k = 5 + 25 + 125 + 625 = 780 < 2014


116

n 4,

for

and

k=1

k=1

sn = 5k 5k = 5 + 25 + 125 + 625 + 3125 = 3905 > 2015


for

n 5.

c.

Every term of this series is between 0 and 1, hence its partial sums increase in

Therefore no

sn

lies between 2014 and 2015.

steps smaller than 1 starting with

s1 = 2999/3000 < 2014.

Moreover,

2999/3000
2999
) =
= 2999 > 2015
(
1 2999/3000
n=1 3000
by the geometric series sum formula.
satisfying the condition

d.

Hence there is at least one partial sum

2014 sn 2015.

The same reasoning as in part

sn

(c) works, this time with the observation that

1
= > 2015 ,
n=1 n

to give the existence of at least one partial sum

sn 2015.

7.

sn

satisfying the condition

2014

Determine whether each of the following series converges or diverges.

a.
d.
g.

n=1

nn

n=1 n!

e.

1
n=3 n ln n ln(ln n)

h.

Solution: a.
Hence

b.

b.

(21/n 21/(n+1) )

We have

lim sn = 1 .

We have

n ln n
n
n=2 3

c.

n sin n
n=1 n +

sin n
2
n=1 n

f.

k=1

k=1

(1)n+1 cos ( )
n
n=1
n
n
5 2
n
n
n=1 7 6

sn = (21/k 21/(k+1) ) = 21/k 21/(k+1) = 2 21/(n+1) .

Therefore the series converges and

k=1

(21/n 21/(n+1) ) = 1 .
n=1

an = n ln n/3n

and

an+1
(n + 1) ln(n + 1)/3n+1 1
n+1
ln(n + 1)
= lim
= lim
lim
n
n an
n
n
n
n ln n/3
3
n
ln n

= lim

ln(x + 1) 1
1/(x + 1) 1
1
1
1 lim
= lim
= 1= .
x
3
ln x
3 x 1/x
3
3
L'H

=
Since

= 1/3 < 1 ,

the series

n ln n
n
n=2 3

converges by the Ratio Test.

117

c.

an = (1)n+1 cos(/n),
lim an = lim cos(/n) = cos 0 = 1 .
n
n
by the nth Term Test.

d.

Since

We have

an = nn /n!

we

have

Therefore

an = cos(/n)
lim an =/ 0 , and

n 2,

for

and

the series diverges

and

an+1
(n + 1)n+1 /(n + 1)!
1
= lim
= lim
= lim (1 + ) = e .
n
n an
n
n
n /n!
n

e.

= e > 1,

Since

nn
n=1 n!

the series

diverges by the Ratio Test.

We observe that

1/(n + n sin n)
L = lim
= lim
n
n
1/n

0<L<

Since

and the harmonic series

n sin n
n=1 n +

f.

diverges, we conclude that the series

L = lim

series

(5/7)n

n=1
(5n 2n )/(7n 6n )

n
n
5 2n

n=1

on

1
n=1 n

diverges by the Limit Comparison Test.

The geometric series

have

g.

1
1
=
=1.
sin n 1 + 0
1+
n

Since

7n 6n
ln x

(1, ).

5n /7n

converges as

r = 5/7 r = 5/7 < 1.

1 (2/5)n 1 0
=
= 1.
n 1 (6/7)n
10

= lim

Since

We also

0 < L < ,

the

converges by the Limit Comparison Test.

(0, ) , ln(ln x) is an increasing function


positive on [3, ) . Therefore x ln x ln(ln x) is

is an increasing function on

Both

ln x

ln(ln x) are
[3, ) as it is the product of three positive and increasing
implies that 1/(x ln x ln(ln x)) is a positive and decreasing

and

positive and increasing on


functions. This in turn
function on

[3, ) .

Since it is also continuous, we can apply the Integral Test. The

improper integral

dx
du
=
x ln x ln(ln x)
ln(ln 3) u

diverges, where we used the change of variable

the series

h.
0

n=3 n ln n ln(ln n)

1 + sin n
2
2
2
n
n

for

Hence

diverges.

Consider the series

u = ln(lnx), du = dx/(x ln x) .

1 + sin n
.
n2
n=1

n 1.

The

Since

0 1 + sin n 2

1
2
n=1 n

p-series
118

converges as

for

n 1,

p = 2 > 1.

we have

Therefore

the series

2
1
=
2

2
2
n=1 n
n=1 n

Direct Comparison Test.

sin n
,
2
n=1 n

Remark:

1 + sin n
converges by the
n2
n=1

1 + sin n
1
and
converge, the series

2
2
n
n=1
n=1 n

converges. Then the series

Since both

which is their dierence, also converges.

Other tests can be used too. Here are some examples:

1
2
n=1 n

In part

(a), the Limit Comparison Test with the series

In part

(b), the Limit Comparison Test with the geometric series

In part

(d), the Root Test also works where we use the fact that n
lim

119

also works.

1
n
n=1 2

also works.

1/n

(n!)
n

1
e

Remark:

The following was a bonus problem on Moodle in Spring 2010 Math 102 course.

Problem K: In class we showed that 0.12 = 0.121212. . .

= 12/99 as an application

of the geometric series.

x = 0.121212 . . . , then 100x =


therefore x = 12/99 . Of course

We learn the following trick in elementary school: If

12.121212 . . . ;

their dierence gives

99x = 12

and

when we see this in elementary school, no one talks about convergence.


That is what we are going to do in this problem. We will forget about convergence
as we know it, and take a trip to Tersonia.
In Tersonia after they teach the students about integers and rational numbers,
they come to the decimal representations of numbers. As you know our decimal
expansions have the form

dn dn1 . . . d2 d1 d0 .d1 d2 . . .
where each

di

is in

{0, 1, . . . , 9}.

We can have innitely many nonzero digits after

the decimal point, but we must have only nitely many nonzero digits before the
decimal point. In Tersonia they do just the opposite. Their decimal expansions
have the form

. . . t3 t2 t1 t0 .t1 t2 . . . tn
where each

ti

is in

{0, 1, . . . , 9}.

Note that there is no minus sign. They can have

innitely many nonzero digits before the decimal point, but they can only have
nitely many digits after the decimal point.
Take a few minutes to convince yourself that Tersonians can add and multiply
their decimal expansions just like we do.
Why no minus sign? Well, because Tersonians don't need it. Negative numbers
are already there.
Then

y = 12.0 = . . . 121212.0 .
y = 12/99 . So in fact 12.0

For instance, consider the number

100y = . . . 121200.0

and

99y = 12 .

Therefore

is a negative number.
Here are some problems from Tersonian Elementary School Mathematics Book :

Part

(e)

a.

1
=?
2

e.

Find two nonzero numbers

b.

1
=?
3
A

and

c.

1
=?
7

such that

was later turned into a programming challenge.

d.
A B = 0.

A Java applet that computes

last n digits of A and B when their last digits are


http://www.fen.bilkent.edu.tr/otekman/calc2/ters.html .

the

120

1 = ?

given

can

be

found

at

8.

Consider

xn
n
2
n=0 5 (n + 1)

f (x) =

a.

Show that

b.

Show that

4
3
< f (3) < .
3
2
3
4
< f (3) < .
4
5

Solution: a.

We have:

3n
3
32
343 4
>
1
+
+
=
>
n
2
2
5 2 5 5 250 3
n=0 5 (n + 1)

f (3) =
and:

3n
3
3n
=
1
+
+

n
2
5 2 n=2 5n (n2 + 1)
n=0 5 (n + 1)
13 1 3n 13 1 (3/5)2 37 3
<
+
=
+
=
<
10 5 n=2 5n 10 5 1 3/5 25 2

f (3) =

b.

We have

f (3) = (1)n
n=0
and

f (3) = (1)n
n=0

3
32
193 4
3n
<
1

+
=
<
n
2
2
5 (n + 1)
5 2 5 5 250 5

3n
3
32
33
469 3
>
1

=
>
n
2
2
3
5 (n + 1)
5 2 5 5 5 10 625 4
n

1
3
}
by the Alternate Series Estimate as {( )
2
5
n + 1 n=0
n
3
1
lim ( ) 2
= 0.
n 5
n +1

121

is a decreasing sequence and

9.

Determine whether the sum of the series

Solution:

Let

bn =

4n
n!(n + 1)!

for

(4)n
n=0 n!(n + 1)!

n 0.

is positive or negative.

Then:

b >0 n0
lim b = lim n!(n4+ 1)! = lim 4n! lim (n +1 1)! = 0
for all

as both limits are 0, the rst

one being one of the  Useful Limits .

n bn+1

4n
4n+1

(n + 2)(n + 1) 4 n 1.
n!(n + 1)! (n + 1)!(n + 2)!

Therefore the series satises the conditions of the Alternating Series Test.

In

particular, it converges. Moreover,

(4)n
4
42
43
44
4 4 4
=1
+

+
=12+ +

1!2! 2!3! 3!4! 4!5!


3 9 45
n=0 n!(n + 1)!

S=

s3 =1/9

and, by the Alternating Series Estimate,

1/45,

Remark:

and

In other words,

S (1/9) <

is negative.

(4)n
n=0 n!(n + 1)!

S s3 < b4 .

b4

is

1
J1 (4),
2

(1)n (x/2)2n+1
is the
n!(n + 1)!
n=0
approximately 0.03302166401.

where

the rst kind of order 1, and its value is

J1 (x) =

122

Bessel function of

10.

Find the radius of convergence of the power series

(1)n
n=0

Solution:

xn
.
(2n + 1)(n2 + 1)

We will give two dierent solutions.

Solution 1 : Here we will use the Ratio Test.


We have

an = (1)n

xn
(2n + 1)(n2 + 1)
= lim

and

an+1
an
(1)n+1

= lim

xn+1

(2n+1 + 1)((n + 1)2 + 1)

(1)n

xn

(2n + 1)(n2 + 1)

xn+1
(2n+1 + 1)((n + 1)2 + 1)
= lim
n
xn
(2n + 1)(n2 + 1)
1 + 2n
n2 + 1
= lim (

)x
n 2 + 2n (n + 1)2 + 1
x
=
.
2
If

x < 2 ,

then

= x/2 < 1

Test. On the other hand, if

and the power series converges absolutely by the Ratio

x > 2 ,

then

= x/2 > 1

and the power series diverges

by the Ratio Test.


It follows by the denition of the radius of convergence that

R = 2.

Solution 3 : Here we will use the radius of convergence formulas.

We have

cn =

(1)n
,
(2n + 1)(n2 + 1)

and the radius of convergence formula gives

cn+1
1/((2n+1 + 1)((n + 1)2 + 1))
1
= lim
= lim
n
R n cn
1/((2n + 1)(n2 + 1))
n2 + 1
1 + 2n
1

= lim (
)
=
.
n 2 + 2n (n + 1)2 + 1
2
Therefore

R = 2.

123

11.

Consider the power series

(1)n
n=0

a.

x2n+1
(2n + 1)(n2 + 1)

Find the radius of convergence of the power series.

b.

Determine whether the power series converges or diverges at the right endpoint of its

interval of convergence.

c.

Determine whether the power series converges or diverges at the left endpoint of its

interval of convergence.

Solution: a.

We have

an = (1)n

x2n+1
(2n + 1)(n2 + 1)

and

= lim an 1/n
n

1/n

x2n+1
= lim ( n
)
n (2 + 1)(n2 + 1)
x2 x1/n
= lim
n 2 (1 + 2n )1/n n1/n (1 + n2 )1/n
2

=
If

x <

2,

Root Test;
Root Test.

b.

At

x=

x
.
2

2
= x
/2 < 1 and the power series
and if x >
2
, then = x2 /2 > 1 and the
Therefore R =
2.
then

converges absolutely by the


power series diverges by the

we have

2n+1
x
(
2)2n+1
n
=
(1)
(1)n n

(2 + 1)(n2 + 1) n=0
(2n + 1)(n2 + 1)
n=0

2n
= 2 (1)n n
.
(2 + 1)(n2 + 1)
n=0

Consider

the

corresponding

absolute

value

series

2n
.
Since
n
2
n=0 (2 + 1)(n + 1)

1
p-series 2 with p = 2 > 1
n=1 n

2n
1
for all n 1 and the
<
(2n + 1)(n2 + 1) n2

2n
converges,
converges by the Direct Comparison Test; and then
n
2
n=0 (2 + 1)(n
+ 1)
the power series at x =
2 converges absolutely by the Absolute Convergence Test.
0<

c.

At

x= 2

we have

x2n+1
( 2)2n+1
n
= (1) n
(1) n
(2 + 1)(n2 + 1) n=0
(2 + 1)(n2 + 1)
n=0

2n
= 2 (1)n+1 n
.
(2 + 1)(n2 + 1)
n=0

124

This is just

1 times the series we considered in part (b), and therefore it converges

absolutely.

12.

Determine the radius of convergence and the interval of convergence of the power series

xn
.
n+1
n=0 3n + (1)

Also determine the type of convergence at each point of the interval of convergence.

Solution:

As

cn =

1
3n + (1)n+1

, the formulas for the radius of convergence gives

cn+1
1/(3(n + 1) + (1)n+2 )
3 + (1)n+1 /n
1
= lim
= lim
=
lim
= 1.
R
cn
1/(3n + (1)n+1 )
3 + 3/n + (1)n+2 /n
Therefore

x > 1.
At

x=1

R = 1.

Hence we have absolute convergence for

we have

x < 1

and divergence for

xn
1
=
.

n+1
n+1
n=0 3n + (1)
n=0 3n + (1)

Since

1/(3n + (1)n+1 )
1
1
= lim
=
n
n 3 + (1)n+1 /n
1/n
3

L = lim

is a positive real number and the harmonic series


diverges by the Limit Comparison Test at
At

x = 1

we have

x = 1.

1
n=1 n

diverges, the power series

xn
(1)n
=
.

n+1
n+1
n=0 3n + (1)
n=0 3n + (1)

Let

i.
ii.

iii.

un =

1
3n + (1)n+1

1
> 0 for all n 1 .
3n + (1)n+1
0 < 3n + (1)n+1 3n + 1 < 3n + 2 3(n + 1) + (1)n+2
un > un+1 for n 1 .
1
= 0.
lim un = lim
n
n 3n + (1)n+1
un =

for all

n 1.

Hence

It follows that the series

(1)n
n+1
n=0 3n + (1)

converges by the Alternating Series Test.

Its absolute value series is the same as the series

1
n+1
n=1 3n + (1)

1+

and we showed

that this series diverges. Hence the power series converges conditionally at

125

x = 1 .

R = 1,

To summarize, the radius of convergence is


is

[1, 1),

converges conditionally at

13.

the interval of convergence

the power series converges absolutely at every point of

x = 1 .

(1, 1),

and it

Consider the power series

a.

1 3 5 (2n 1) n
x .
n=1 2 4 6 (2n)

f (x) = 1 +

Show that the radius of convergence of the power series is

b.

R = 1.

Determine the behavior of this power series at the endpoints of its interval of

convergence.

c.

Show that

d.

Solve this dierential equation to show that

e.

Show that

2(1 x)f (x) = f (x)

for

x < 1.
f (x) =

1
1x

for

x < 1.

1 3 5 (2n 1) x2n+1

2n + 1
n=1 2 4 6 (2n)

arcsin x = x +
for

x < 1.

Solution: a.

We have

cn =

1 3 5 (2n 1)
2 4 6 (2n)

for

n 1.

We use the radius of

convergence formula

1 3 5 (2n 1)(2n + 1)
1
cn+1
2n + 1
2 4 6 (2n)(2n + 2)
= lim
= lim
= lim
=1
n
n 2n + 2
1 3 5 (2n 1)
R n cn
2 4 6 (2n)
to obtain

b.

At

R = 1.

x=1

we have the series

1 3 5 (2n 1)
n=1 2 4 6 (2n)

1+

. Since

1 3 5 (2n 1) 3 5
2n 1 1
1
=

>
2 4 6 (2n)
2 4
2n 2 2n 2n

for

1
n=1 n

n > 1 and the harmonic series

diverges, we conclude that the series at

x=1

diverges by the Direct Comparison Test.

At

x = 1

un =

we obtain the alternating series

1 3 5 (2n 1)
.
2 4 6 (2n)

1 + (1)n
n=1

We have

(i ) un > 0

126

for all

1 3 5 (2n 1)
2 4 6 (2n)

n 0,

and we also have

with

(ii )

un > un+1

for all

n0

as

un /un+1 = (2n + 2)/(2n + 1) > 1

for

n 0.

On the other

hand,

12 32 52 (2n 1)2 1 3 3 5
(2n 3)(2n 1) 2n 1 1
1
= 2 2

<
2
2
2
2
2
2 4 6 (2n)
2
4
(2n 2)
2n
2n 2n

for n > 1. Therefore 0 < un < 1/ 2n for n > 1, and the Sandwich Theorem gives
(iii ) lim un = 0 . We conclude that the series at x = 1 converges by the Alternating
u2n =

n
Series Test. The convergence is conditional as we have already seen that the absolute
value series diverges.

c.

For

x < 1

we have

1 3 5 (2n 1) n
x .
n=1 2 4 6 (2n)

f (x) = 1 +
Dierentiating this we get

f (x) =
for

x < 1 .

1 1 3 5 (2n 1) n1
+
x
2 n=2 2 4 6 (2n 2) 2

Therefore

1 3 5 (2n 1) n
x
n=2 2 4 6 (2n 2)

2xf (x) = x +
and

2f (x) = 1 +
for

x < 1 .

3
1 3 5 (2n + 1) n
x+
x
2
n=2 2 4 6 (2n)

Taking the dierence of these two, we obtain

1
1 3 5 (2n 1) 2n + 1
x+
(
1) xn
2
2n
n=2 2 4 6 (2n 2)

1 3 5 (2n 1) 1 n
1
=1+ x+

x
2
n=2 2 4 6 (2n 2) 2n

1
1 3 5 (2n 1) n
x
=1+ x+
2
n=2 2 4 6 (2n)

1 3 5 (2n 1) n
=1+
x
n=1 2 4 6 (2n)
= f (x)

2(1 x)f (x) = 1 +

for

x < 1.

127

d.

For

x < 1,
f (x)
1
=
f (x) 2(1 x)
1
ln f (x) = ln 1 x + C for some constant C
2
A
f (x) =
for some constant A .
1x

2(1 x)f (x) = f (x)

Now substituting

e.

x=0

gives

We have

for

x < 1 .

Substituting

for

x2 < 1 .

1 = f (0) = A.

Therefore,

f (x) =

1
1x

for

x < 1.

1
1 3 5 (2n 1) n
=1+
x
1x
n=1 2 4 6 (2n)

x2

for

1
1 x2

gives

1 3 5 (2n 1) 2n
x
n=1 2 4 6 (2n)

=1+

Integrating this we obtain

1 3 5 (2n 1) x2n+1

+C
2n + 1
n=1 2 4 6 (2n)

arcsin x = x +
for

x < 1 .

Substituting

x=0

gives

0 = arcsin 0 = C ,

and hence

C = 0.

1 3 5 (2n 1) x2n+1

2n + 1
n=1 2 4 6 (2n)

arcsin x = x +
for

x < 1 .

Remark:

It can be shown that

for

1 x < 1 ,

and

1 3 5 (2n 1) n
1
=1+
x
1x
n=1 2 4 6 (2n)

1 3 5 (2n 1) x2n+1

2n + 1
n=1 2 4 6 (2n)

arcsin x = x +
for

x 1 .

128

Thus

14.

Consider the power series

n(2x 1)3n+1
5n
n=1

a.

Find the radius of convergence of the power series.

b.

Find the sum of the power series explicitly.

Solution: a.

n(2x 1)3n+1
5n

an =

gives

an+1
an
(n + 1)(2x 1)3(n+1)+1 /5n+1
= lim
n
n(2x 1)3n+1 /5n
n + 1 2x 13
= lim

n n
5
2x 13
=
.
5

= lim

< 1 and the series converges by the Ratio Test; and


> 1 and the series diverges by the Ratio Test. Since
3
1
5
2x 13 /5 < 1 x <
, it follows that the radius of convergence of the
2
2

3
5/2 .
power series is R =

If
if

2x 13 /5 < 1 ,
2x 13 /5 > 1 ,

b.

then

then

We know that

xn =
n=0
for

x < 1 .

Dierentiating this we obtain

nxn1 =
n=1
for

x < 1 .

1
1x

Now we replace

with

1
(1 x)2

(2x 1)3 /5

to get

n(2x 1)3(n1)
=
5n1
n=1

for

(2x 1)3 /5 < 1 .

for

x 1/2 <

(2x 1)3
(1
)
5

Finally we multiply by

n(2x 1)3n+1
=
5n
n=1

(2x 1)4 /5

(2x 1)4 /5
2

(1

(2x 1)3
)
5

5/2 .

129

to obtain

(2x 1)4
5

4 (4x3 6x2 + 3x 3)2

15.

Consider the function dened by:

xn

n=0

n! 2n(n1)/2

f (x) =

a.

Find the domain of

b.

Evaluate the limit

c.

Show that

d.

Show that

f.

f (x) ex
x0 1 cos x
3
f (2) < e + .
2
lim

f (2) < 0 .

Solution: a.

Since

cn =

1
n! 2n(n1)/2

, the radius of convergence formula gives

1
cn+1
1/((n + 1)! 2(n+1)n/2 )
1
= lim
= lim
= lim
=0.
n(n1)/2
n
n (n + 1)2n
R n cn
1/(n! 2
)
Therefore

b.

R = .

This means that the domain of

Using

xn

n=0

n! 2n(n1)/2

f (x) =

=1+x+

is

(, ) .

x2
x3
+ 3
+
2 2! 2 3!

we obtain

f (x) ex
= lim
x0 1 cos x
x0

lim

c.

(1 + x +

x2
x3
x2 x3
+ 3
+ ) (1 + x +
+
+ )
2 2! 2 3!
2! 3!

x2 x4
+
)
2! 4!
1
1 7
7 3
x2

x
x
4
48
4
48
= lim
= lim
1 4
1 2
x0 1
x0 1
x2
x +

x +
2
24
2 24
1

1
= 4 = .
1
2
2
1 (1

We have

2n

n=0

n! 2n(n1)/2

f (2) =

On the other hand,

n=0

n! 2n(n3)/2

=1+2+

2 1
1
.
+ +
n(n3)/2
2! 3! n=4 n! 2

1 1 1
1
=1+1+ + +
.
2! 3! n=4 n!
n=0 n!

e=

130

Since

n(n 3) > 0

n=4

n! 2n(n3)/2

d.

for

n 4,

1
n=4 n!

<

. Therefore,

we have

n! 2n(n3)/2
3
f (2) e < .
2

<

1
n!

for

n 4.

Hence

This time we have

(2)n
n(n1)/2
n=0 n! 2

(1)n
=
n(n3)/2
n=0 n! 2
2
(1)n
=12+ +
2! n=3 n! 2n(n3)/2

(1)n
=
n(n3)/2
n=3 n! 2
1
1
1
1
= + 2
5
+ 9
.
3! 2 4! 2 5! 2 6!

f (2) =

As

(n + 1)2n1 > 1 for n 3 , we have

too. Therefore,

16.

Estimate

f (2) <

x
0 e dx

Solution:

1
n! 2n(n3)/2

1
1
5
+ 2
= < 0.
3! 2 4!
32

with error less than

We have

ex = 1 + x +
for all

x.

x,

i.
ii.

iii.

un =

< 0 for n 3

0.01.

x2
xn
++
+
2!
n!

x4
x2n
+ + (1)n
+
2!
n!

and integration gives

Let

1
(n + 1)! 2(n+1)(n2)/2

Therefore,

ex = 1 x2 +
for all

ex dx = 2

22n+1
.
n! (2n + 1)

23 25
22n+1
+
+ + (1)n
+.
3 2! 5
n! (2n + 1)

Then:

22n+1
> 0 for all n 0 .
n! (2n + 1)
2n2 3n 1 > 0 for n 2 (n + 1)(2n + 1) > 22 (2n + 1)
un > un+1 for n 2 .
4n
2
lim un = lim (
) = 0.
n
n n! 2n + 1
un =

131

for

n 2.

Hence

The series satises the conditions of Alternating Series Test for

11!) = 32768/3586275 0.009 < 0.01,

n 2.

Since

it follows by the Alternating Series Estimate

that the sum

ex dx 2
2

approximates

17.

219
221
12223758182
23 25
+
+
+
=
0.89
3 2! 5
9! 19 10! 21 13749310575

x
0 e dx
2

with error less than

0.01.

Find the exact value of

Solution:

1
n
n=0 4 (2n + 1)

On one hand we have

1
(1/4)n (1/2)2n
(1/2)2n+1
=
=
=
2

n
n=0 2n + 1
n=0 2n + 1
n=0 2n + 1
n=0 4 (2n + 1)
3
5
(1/2)
(1/2)
= 2 (1/2 +
+
+ ) .
3
5

On the other hand we have

ln(1 + x) = (1)n+1
n=1
for

1 < x 1 .

In particular,

x = 1/2

xn
x2 x3 x4 x5
=x
+

n
2
3
4
5
gives

3
1
(1/2)2 (1/2)3 (1/2)4 (1/2)5
ln ( ) = ln (1 + ) = 1/2
+

+
,
2
2
2
3
4
5
and

x = 1/2

gives

1
1
(1/2)2 (1/2)3 (1/2)4 (1/2)5
ln ( ) = ln (1 ) = 1/2

.
2
2
2
3
4
5
Therefore

n=0

4n (2n + 1)

223 /(23

3
1
= ln ( ) ln ( ) = ln 3 .
2
2

132

Part 4: Vector Analysis

1.

Find the value of the line integral

(3x2 y 2 + y) dx + 2x3 y dy
C

where

is the cardioid

Solution:

r = 1 + cos

We use the Green's Theorem

where

parameterized counterclockwise.

M dx + N dy = (
R

N M

) dA
x
y

is the region enclosed by the simple closed curve

C.

Therefore

M
M
N
N


(3x2 y 2 + y) dx + 2x3 y dy = ( ( 2x3 y ) (3x2 y 2 + y)) dA


y
R x
C

= (6x2 y (6x2 y + 1)) dA


R

= dA
=
=

R
2

1+cos

0
2
0

r dr d

r2 r=1+cos
]
d
2 r=0

1 2
= (1 + 2 cos + cos2 ) d
2 0
1 2
1 + cos 2
=
(1 + 2 cos +
) d
2 0
2
1 3
= 2
2 2
3
= .
2

Remark We used the Circulation-Curl Form

of the Green's Theorem, but the computation is

exactly the same with the Flux-Divergence Form :

M dy N dx = (
R

133

M N
+
) dA
x
y

In fact, when expressed in terms of components and coordinates, both forms of the Green's
Theorem can be summarized and most easily remembered as

C
where

= A dx + B dy

Remark following Example 39 in Part 2.

Indeed, if

M
M
N
N
dx +
dy) dx + (
dx +
dy) dy
x
y
x
y

= M dy N dx,

then

d = dM dy dN dx = (
=

M
M
N
N
dx +
dy) dy (
dx +
dy) dx
x
y
x
y

N
M
N
M N
M
dx dy
dy dx =
dx dy +
dx dy = (
+
) dx dy
x
y
x
y
x
y

dx dx = 0 = dy dy
dx dy

now we must always get a

2.

The multiplication of

M
N
M
N
N M
dy dx +
dx dy =
dx dy +
dx dy = (

) dx dy ,
y
x
y
x
x
y

where we used

d = dA dx + dB dy .

then

d = dM dx + dN dy = (

and if

is a dierential form and

dierentials here is as described in the

= M dx + N dy ,

= d

and

dy dx = dx dy .

Unlike in the case of change of variables,

under the double integral and we must not get rid of the sign.

F dr

Evaluate

where

F=
and

y
x
i+ 2
j
2
+ 9y
4x + 9y 2

4x2

is the unit circle parametrized in the counterclockwise direction.

Solution:

Observe that
curl

at all points
Consider

y
( 2
)
( 2
)
2
x 4x + 9y
y 4x + 9y 2
1
8x2
1
18y 2
= 2

=0
4x + 9y 2 (4x2 + 9y 2 )2 4x2 + 9y 2 4x2 + 9y 2

F=

(x, y) =/ (0, 0).

C0 r = (cos t)/2 i + (sin t)/3 j, 0 t 2 , the counterclockwise


4x2 + 9y 2 = 1. Let R be the region lying inside the

parametrization of the ellipse

unit circle and outside this ellipse.

Since curl

F=0

at all points of

Green's Theorem,

C0

Therefore by the generalized

y dx + x dy
2
2
C0 4x + 9y
t=2 (sin t)/3 d((cos t)/2) + (cos t)/2 d((sin t)/3)
=
t=0
cos2 t + sin2 t
1 2
1

=
dt = 2 = .
6 0
6
3

F dr =

F dr =
C

R, curl F dA = 0 .
R

134

3.

Find the surface area of the parameterized torus

r = (a + b cos u) cos v i + (a + b cos u) sin v j + b sin u k ,


0 u 2 , 0 v 2 ,

Solution:

where

0<b<a

are constants.

We have:

ru = b sin u cos v i b sin u sin v j + b cos u k


rv = (a + b cos u) sin v i + (a + b cos u) cos v j
ru rv = b(a + b cos u) cos v cos u i b(a + b cos u) sin v cos u j b(a + b cos u) sin u k
ru rv = b(a + b cos u)(cos2 v cos2 u + sin2 v cos2 u + sin2 u)1/2 = b(a + b cos u)
Therefore
Surface Area

= ru rv dA = b(a + b cos u) dA
=

where

R
2

R = {(u, v) 0 u 2

0
and

b(a + b cos u) du dv = ab 2 2 = 4 2 ab

0 v 2}.

135

4.
x2

Find the area of the portion


+ y 2 + z 2 = 4.

Solution:

of the cylinder

x2 + y 2 = 2y

that lies inside the sphere

F (x, y, z) = x2 + y 2 2y . Then the cylinder is given by F (x, y, z) = 0.


We will use the projection to the yz -plane. This projection is 2-1 from the portion of
the cylinder inside the sphere to the region R given by the inequalities 0 2y 4z 2
in the yz -plane, and by symmetry
Let

Surface Area of

where

p = i.

F
S = 2
dA
R F p

We have

F = 2xi + (2y 2)j


F = (4x2 + (2y 2)2 )1/2 = (4x2 + 4y 2 8y + 4)1/2 = 2
F p = F i = 2x

on

and therefore

Surface Area

F
2
1
dA = 2
dA = 2
dA
R F p
R 2x
R x

1
2y y 2 on S
= 2
dA
as x =
R
2y y 2

2
42y
2
1
4 2y

= 2
dz dy = 2 2
dy
2
0
42y
0
2y y
2y y 2
2 1

= 4 2 dy = 4 2 2 2 = 16 .
y
0
= 2

136

5.

Consider the parametrized surface

the area of the portion of the surface

S r = u2 i + 2 uvj + v 2 k, < u < , 0 v < .


S that lies inside the unit ball x2 + y 2 + z 2 1.

Find

x = u2 ,
y = 2uv and z = v 2 on the surface. Therefore,
x2 + y 2 + z 2 1 means (u2 )2 + ( 2uv)2 + (v 2 )2 1. In other words, (u2 + v 2 )2 1, or
u2 + v 2 1. Hence the part of the surface lying inside the sphere is the image of the
region R, the upper half of the unit disk, in the uv -plane.

Solution:

We have

To compute the surface area we rst compute:

ru = 2ui + 2vj

rv = 2uj + 2vk

ru ru = 2 2v 2 i 4uvj + 2 2u2 k

ru ru = ((2 2v 2 )2 + (4uv)2 + (2 2u2 )2 )1/2 = 2 2(u2 + v 2 )

Therefore,
Surface Area

= ru ru du dv
R

= 2 2 (u2 + v 2 ) du dv

= 2 2

2
0 r r dr d

1
= d
2 0

=
2
where we used the polar coordinates in the

Remark:

uv -plane.

There is a shorter way of solving this problem which does not use Calculus. The

given parametrization maps the upper half-uv -plane onto the half-cone given by the equation

y 2 = 2xz ,

and the conditions

x0

and

z0

u-axis).
positive x- and

in a one-to-one manner (except on the

This half-cone has its vertex at the origin, its axis lies along the bisector of the
z -axes, and it has an opening angle of 45 . The portion of this half-cone cut o by the unit
sphere is the lateral surface of a right cone with slant height
with area

r = / 2.
137

=1

and radius

r = 1/ 2,

hence

6.

Verify Stokes's Theorem for the vector eld F = y i + z k and the surface S , where S is the
z = x2 +y 2 satisfying z 3, with the unit normal vector eld n pointing

portion of the paraboloid


away from the

z -axis.

Solution:

is bounded by the curve

=
=

t=2
t=0
2
0

Cr=

3 cos t i

3 sin t j + 3 k, 0 t 2 .
Note that this parametrization is consistent with the direction of n. The circulation
of F around C is

F dr = (y i + z k) (dx i + dy j + dz k)
S

( 3 sin t i + 3 k) (d( 3 cos t) i + d( 3 sin t) j + d(3) k)

( 3 sin t)( 3 sin t) dt = 3

2
0

sin2 t dt = 3

f (x, y, z) = z x2 y 2 , and
f = 2x i 2y j + k points in the opposite direction to n, so we will choose the
minus sign from in the ux integral. The projection of S into the xy -plane is the
disk R = {(x, y) x2 + y 2 3}. Finally,
RRR i
j
k RRRR
RR
RR
RR
RRR
RRR
RR
RR

RRR = k .
F = RRR
RR x y z RRR
RRR
RRR
RR
RR
RRR
R
0
z RRRR
RR y
The ux of F across S is
f
S ( F) n d = R F k f dA
On the other hand, the paraboloid is a level surface of

= (k) (2x i + 2y j k) dA
R

= dA = Area of R = ( 3)2 = 3 .
R

Hence

S ( F) n d =

F dr.
C

7.

Verify Divergence Theorem for the vector eld


{(x, y, z) x2 + y 2 + z 2 4} .

Solution:

F = xz i + yz j + z 3 k

and the region

F = (xz)/x + (yz)/y + (z 3 )/z = z + z + 3z 2 = 2z + 3z 2 ,

2
D F dV = D (2z + 3z ) dV

2
0

2
2
2
0 0 (2 cos + 3 cos ) sin d d d

96
cos2 ) sin d d
(8 cos +

5
0
0
64 2
128
=
d =
.

5 0
5

138

and

D =

S = {(x, y, z) x2 + y 2 + z 2 = 4}, we
divide it as the upper hemisphere S1 = {(x, y, z) x2 + y 2 + z 2 = 4 and x 0} and the
lower hemisphere S2 = {(x, y, z) x2 + y 2 + z 2 = 4 and x 0}, and project both onto
the disk R = {(x, y) x2 + y 2 4} in the xy -plane.

To compute the outward ux through the sphere

The sphere is a level surface of


in the same direction as

n,

f (x, y, z) = x2 + y 2 + z 2 . f = 2x i + 2y j + 2z k

the outward pointing unit normal vector eld on

we will choose the plus sign from

in the ux integral.

For the upper hemisphere we have

f
S F n d = R F k f dA
1

= (xz i + yz j + z 3 k)
R

2x i + 2y j + 2z k
dA
2z

= (x2 + y 2 + z 3 ) dA
R

= (x2 + y 2 + (4 x2 y 2 )3/2 ) dA
=
=

R
2

2
2 3/2
0 (r + (4 r ) ) r dr d

0
2
0

(4 +

32
64
) d = 8 +
.
5
5

A similar computation for the lower hemisphere gives

64
S F n d = 8 + 5 .
2
Therefore

S F n d = S F n d + S F n d
2

64
64
128
= (8 +
) + (8 +
)=
,
5
5
5
and

points

S F n d = D F dV

139

S,

so

You might also like